You are on page 1of 77

Quiz no.

1 Statutory Merger and Consolidation


Question 1

On January 1, 2020 Entity A acquired the net assets of Entity B by issuing common stocks with fair value of P150,000 and par value
of P100,000. Aside from that, Entity A is required to pay P132,000 cash to the owners of Entity B on December 31, 2020. The
applicable effective interest rate of this contingent consideration is 10%. On January 1, 2020, the net assets of Entity B is reported
at a book value of P250,000. On the acquisition date, all assets and liabilities of Entity B are property valued except for the inventory
that is undervalued by P20,000 and the note payable that is overvalued by P30,000. As a result of acquisition, Entity A incurred and
paid the following costs: (1) acquisition related cost of P20,000; (2) indirect cost of acquisition of P10,000; and (3) stock issuance
costs of P30,000.

What is the goodwill or gain on bargain purchase arising from business combination to be recognized by Entity A on January 1,
2020?

30,000 gain on bargain purchase

Question 2

Rhian acquires 100 percent of the outstanding voting shares of Franco Company on January 1, 2019. To obtain these shares,
Rhian pays P2,000,000 in cash and issues 100,000 shares of its owns P10 par value common stock. On this date, Rhian's stock
has a fair market value of P18 per share. Rhian also pays P100,000 to a local investment company for arranging the acquisition. An
additional P60,000 was paid by Rhian in stock issuance costs

The book value for both Rhian and Franco as of January 1, 2019, follow. The fair market value of each of Franco's accounts is also
included. In addition, Franco holds a fully amortized patent that still retains a P300,000 value.

Franco Company

Rhian, Inc. Book Fair Market

Book value value value

Cash P3,000,000 P 400,000 P 400,000

Receivables 1,600,000 900,000 800,000

Inventory 2,200,000 1,300,000 1,300,000

Land 1,000,000 600,000 600,000

Buildings (net) 4,000,000 1,100,000 1,400,000

Equipment (net) 1,200,000 500,000 500,000

Accounts payable 1,600,000 300,000 300,000

Long-term liabilities 3,800,000 1,700,000 1,500,000

Common stock 4,000,000 400,000

Retained earnings 3,600,000 2,400,000


Assume that the business combination is a statutory merger so that Franco's accounts are to be transferred to the records of Rhian
with Franco subsequently being dissolved as a legal corporation.

How much goodwill must be recognized as a result of this business combination?

P300,000

Question 3

1 / 1 pts

Where the acquirer purchases assets and assumes liabilities of another entity it does NOT need to consider measurement of:

goodwill

carrying amounts of identifiable net assets

consideration transferred

fair values of identifiable net assets

Question 4

Assume that on April 1, 2019, YGCI agrees to issue 30 million ordinary shares as consideration for the acquisition of 100% of the
issued shares of TLI and the agreement states that control passes as of the date of the agreement (April 1, 2019). YGCI issues the
30 million YGCI shares on June 1. 2019. Which date would be adjudged the acquisition date under PFRS 3?

June 1, 2019

Question 5

If the initial accounting for a business combination is incomplete by the end of the reporting period in which the combination occurs.
The acquirer shall report in its financial statements provisional amounts for the items for which the accounting is incomplete. What is
the maximum term or period of the measurement period?
One year or 12 months from the acquisition date.

1 month from the acquisition date

6 months from the acquisition date

3 months from the acquisition date

Question 6

How shall an acquirer in a business combination account for the changes in fair value contingent consideration classified as
financial liability if the changes result from events after the acquisition date?

The changes in fair value of contingent consideration classified as financial liability shall be retrospectively restated to beginning
retained earnings because they are prior period error.

The changes in fair value of contingent consideration classified as financial liability shall be recognized as gain or loss in profit or
loss because they are not measurement period adjustments.

The changes in fair value of contingent consideration classified as financial liability shall be retroactively adjusted to goodwill/gain on
bargain purchase because they are measurement period adjustments.

The changes in fair value of contingent consideration classified as financial liability shall be retroactively applied to beginning
retained earnings because they are change in accounting policy.

Question 7

Which of the following statements is false?

I. A poison pill is the term used to describe the issuance of a special kind of convertible preference share to deter the acquisition of
the company.

II. The sale of the crown jewels defensive maneuver involves the sale of more assets than does the scorched earth defense.

III. The fat man defensive maneuver involved the acquisition of assets by the potential acquiree.

IV. Golden parachutes give a bonus to all employees if the company is acquired.

V. The packman defensive maneuver is where a potential attempts to purchase the acquirer.
Only I, II, and IV are false

Question 8

The Natural Company acquired 80% of The Loco Company for a consideration transferred of P100 million. The consideration was estimated to
include a control premium of P24 million. Loco's net assets were P85 million at the acquisition date. Are the followingstatements true or
false, according to PFRS 3 Business combinations?

(1) Goodwill should be measured at P32 million if the non-controlling interest is measured at its share of Local's net assets.

(2) Goodwill should be measured at P34 million if the non-controlling interest is measured at fair value.

True, True

Question 9

Which of the following accounting treatments for costs related to business combination is incorrect?

Acquisition related cost such as finder’s fees advisory, legal, accounting. Valuation and other professional and consulting fees and
general administrative costs. Including the costs of maintain ab internal acquisition department shall be recognized as expense in
the profit/loss in the periods in which the costs are incurred.

The costs related to issuance of stocks or equity securities shall be deducted/debited from any share premium from the issue and
any excess is charged to ‘share issuance costs’ reported as contract-equity account against either (1) share premium from other
share issuances or (2) retained earnings.

The costs related to issuance of financial liability at fair value through profit or loss shall recognized as expense while those related
to issuance of financial liability at amortized costs shall be recognized as deduction from the book value of financial liability or treater
as discount on financial liability to be amortized using effective interest method.

The costs related to the organization of the newly formed corporation also known as pre-incorporation costs shall be capitalized as
goodwill or deduction from gain on bargain purchase.

Question 10

Applying acquisition method for business combination requires the following steps. Except

Using equity method

Recognizing and measuring the identifiable assets acquired, the liabilities assumed and any noncontrolling interest in the acquire

Determining the acquisition date


Recognizing and measuring goodwill or an gain from a bargain purchase

Identifying the acquirer.

Question 11

How shall the acquirer account for its previously held equity interest in the acquiree upon obtaining control of the acquiree or how
shall an acquirer account for the business combination achieve in stages a.k.a step acquisition?

The acquirer shall not include the previous held equity interest in the computation of goodwill or gain on bargain purchase arising
from business combination.

The acquirer shall account the transaction as prior period error to be treated by retroactive restatement.

The acquirer shall remeasure its previously held equity interest in the acquiree at its acquisition-date fair value and recognize the
resulting gain or loss in profit/loss.

The acquirer shall treat the transaction as change in accounting policy to be treated retrospectively at acquisition date.

Question 12

Which of the following statements concerning the identification of the acquirer in business combination is incorrect?

In business combination through consolidation. The acquirer is the newly formed corporation.

In some business combination, commonly called “reverse acquisition” the issuing entity is the acquiree while the other entity that
receives the issued shares is the acquirer.

In business combination through merger, the acquirer is the absorbed corporation after the business combination.

In business combination affected primarily by transferring assets or by incurring liabilities or issuing shares, the acquire is usually
the entity that transfers the cash, incurs the liabilities or issues the shares.

Question 13

Some changes in the fair value of contingent consideration that the acquirer recognizes after the acquisition date may be the result
of additional information that the acquirer obtained after that date about facts and circumstances that existed at the acquisition date.
These are called measurement period adjustments that can be adjusted during the measurement period. Which of the following
transactions is considered as a measurement period adjustment that the acquirer shall retrospectively adjust to goodwill/(gain on
bargain purchase) during the measurement period which shall not exceed one year from the acquisition date?

Change in the carrying amount of the financial liability at amortized cost issued as consideration for business combination due to
amortization of the premium/(discount) on financial liability.

Changes in the provisional amount of contingent liability or contingent consideration as a result of new information obtained about
the fats and circumstance that existed as of the acquisition date and, if known, would have affected the measurement of the
amounts recognized as of that date.

Changes in the value contingent consideration occurring within one year from the acquisition date as a result of event occurring
after the acquisition date such as meeting an earnings target, a specified share price or reaching milestone on a research and
development project.

Increase in the fair value of the financial liability at fair value through profit or loss issued as consideration for business combination
due to movement of prices in the exchange market.

Question 14

Company A (Kravis Roberts and Co.) has made an offer to purchase all of the outstanding shares of Company B (RJR Nabisco) for
P10 per share (the current market value of the shares). In response to Company A’s offer, the shareholders of Company B were
given rights to purchase additional shares at P8 per share. Which of the following tactics was employed by Company B to prevent
Company A from acquiring control of Company B?

A reverse take over

Poison pill

Provide a levee to prevent the barbarians to enter the gate

Scorched earth defense

Question 15

Entity A acquired 80,000 out of 100,000 outstanding ordinary shares of Entity B which enabled the former to obtain control of the
latter at an acquisition price of P1,000,000. Entity A paid P100,000 acquisition related costs and P50,000 indirect costs of business
combination.

At the date of acquisition, the net assets of Entity B are reported at P1,600,000. An asset of Entity B is overvalued by P60,000 while
one liability is undervalued by P40,000.
What is the goodwill or gain on bargain purchase arising from business combination?

250,000 gain on bargain purchase

Question 16

On January 1. 20x4, Park Corporation and Strand Corporation and their condensed balance sheet are as follows:

Park Corp. Strand Corp.

Current Assets P70,000 P20,000

Non-current Assets 90,000 40,000

. Total Assets P160,000 P60,000

P10,000
Current Liabilities P30,000

Long-term Debt 50,000

Stockholders' Equity 80,000 50,000

Total Liabilities and Equities P160,000 P60,000

On January 2, 20x4, Park Corporation borrowed P60,000 and used the proceeds to obtain 80% of the outstanding common shares
of Strand Corporation. The P60,000 debt is payable in 10 equal annual principal payments, plus interest, beginning December 31,
20x4. The excess fair value of the investment over the underlying book value of the acquired net assets is allocated to inventory
(60%) and to goodwill (40%).

On a consolidated balance sheet as of January 2, 20x4. what should be the amount for each of the following? Current assets
should be: (2) Non-current asset using proportionate basis (partial) in computing goodwill should be: (3) Stockholders' equity using
full fair value (full/gross-up goodwill) proportionate basis of determine non-controlling interest should be:

(1) P105,000: (2) P138,000; (3) P95,000

Question 17

It is that portion of the profit or loss and net assets of a subsidiary attributable to equity interests that are not owned directly by the
parent.

Controlling interest
Residual interest

Subsidiary interest

Minority interest

Question 18

Cat, Ltd. and Leona, Ltd. exchanged shares in a business combination. After the share exchange, each company held the same
number of voting shares. Which of the following statements is true?

The companies must ask the courts to decide which company is the acquirer.

A number of factors must be considered to determine which company is the acquire

The company with the highest net assets is considered the acquirer.

There is no acquirer as this is not a proper business combination.

Question 19

In a business acquisition, consideration transferred includes which of the following?

I. The fair value of assets transferred by the acquirer.

II. The fair value of the liabilities incurred by the acquirer.

III. The fair value of contingent consideration transferred by the acquirer.

IV. The fair value of the equity interests issued by the acquirer as a part of the acquisition.

V. The fair value of sharebased payments voluntarily ex changed for outstanding sharebased payment awards of the acquiree.

All of the above


Question 20

During the current year, an entity acquired another entity in a transaction properly accounted for as business combination. At the
time of acquisition, some of the information for valuing assets was incomplete. How should the acquirer account for the incomplete
information in preparing financial statements immediately after acquisition?

Record the uncertain items at a provisional amount measured at the date of acquisition.

Record a contra account to the investment account for the amount involved.

Do not record uncertain items until complete information is available.

Record the uncertain items at the carrying amount of the acquiree.

Question 21

Under IFRS 3, contrary to IAS 37, what is the recognition principle of contingent liability assumed in a business combination?

The acquirer shall recognize a contingent liability assumed in a business combination at the acquisition date only if it is virtually
certain that an outflow of resources embodying economic benefits will be acquired to settle the obligation.
The acquirer shall recognize as of the acquisition date a contingent liability assumed in a business combination if it is a present
obligation that arises from past events and its fair value can be measured reliably even only reasonable possible.

The acquirer shall recognize a contingent liability assumed in a business combination at the acquisition date only if it is probable
that an outflow of resources embodying economic benefits will be acquired to settle the obligation.

The acquirer shall recognize a contingent liability assumed in a business combination at the acquisition date only if it is remote that
an outflow of resources embodying economic benefits will be required to settle the obligation.

Question 22

Minority interests shall be presented in the consolidated balance sheet

As noncurrent liability
Separately from liabilities and the parent stockholders' equity
Within equity, separately from the parent shareholders' equity
As component of the parent stockholder’s equity
Question 23

Which statement is correct concerning the treatment of goodwill arising from a business combination?

Goodwill is not recognized as an asset

Goodwill is carried at cost less any accumulated impairment losses

Goodwill is carried at cost less accumulated amortization and any accumulated impairment losses.

Goodwill is carried at cost without amortization and impairment

Question 24

Under which set of circumstances would it not be appropriate to assume the value of the non-controlling interest is the same as the
controlling shares?

The acquisition is for less than 100% of the subsidiary.


Active market prices for shares not obtained by the acquirer imply a different value.
The amount of the “control premium” cannot be determined.
The fair value of the non-controlling shares can be inferred from the value implied by the acquisition price.

Question 25

Which of the following is not a reason for provisional accounting for a business combination?

some of the consideration is deferred until sometime after the business combination
fair value of the contingent liabilities cannot be measured reliably until sometime after the business combination
fair value of assets acquired cannot be conclusively measured until sometime after the business combination
some of the costs can only be determined on a provisional basis until sometime after the business combination
Question 26

On July 1, 2019, Pasay Company, an SME, paid P755,000 cash for the net assets of Stir Company. The recorded assets and
liabilities of Stir are: Cash, P74,000; Inventory, P215,000; Land, P200,000; Buildings (net), P208,000; and liabilities, P220,000. At
the same date, Stir's inventory had a fair value of P184,000; the land, P271,500; and the building (net) P187, 500. Pasay Company
incurred direct costs of P55, 000 and indirect expenses of P20,000.

Determine the amount of goodwill resulting from the business combination on December 31, 2019.

P 313, 000

297,350

P281, 700

P258, 000

Question 27

A subsidiary shall be excluded from consolidation when

I. Its business activities are dissimilar from those of the enterprise within the group.

II. Control is intended to be temporary because the subsidiary is. acquired and held exclusively with a view to its disposal within 12
months from acquisition.

III. It operates under severe long-term restrictions which significantly impair its ability to transfer funds to the parent but the parent
continues to control the subsidiary.

Il and III

Question 28

Great Company has gained control over the operations of Superb Corporation by acquiring 85% of its outstanding capital stock for
P15,480,000. This amount includes a control premium of P180,000. Acquisition expenses, direct and indirect, amounted to,
P495,000 and P252,000 respectively.

GREAT SUPERB
Book Value Book Value

Cash P21,249,000 768,000

Accounts Receivable 1,800,000 1,950,000

Inventories 3.300,000 2,160,000

Prepaid Expenses 891,000 750,000

Land 14,100,000 5,274,000

Building 9,360,000 3,348,000

Equipment 1.800,000 1,110,000

Goodwill - 1,800,000

Total Assets 52,500,000 17,160,000

Accounts Payable 4,050,000 1,518,000

Notes Payable 8,400,000 4,380,000

Ordinary shares, 50 par 20,400,000 4,800,000

Share premium 9,450,000 3,600,000

Retained earnings 10,200,000 2,862,000

Total equities P52,500,000 P17,160,000

The following were ascertained on the date of acquisition for the Acquired Corporation:

 The value of receivables and equipment has decreased by P150,000 and P84,000 respectively.

 The fair value of inventories are now P2,616,000 whereas the value of land and building have increased by P2,826,000 and
P642,000, respectively.

 There was an unrecorded accounts payable amounting to P162,000 and the fair value of notes is P4,428,000.

Compute for the following balances to be presented in the consolidated statement of financial position

on the date of business combination:


Total Assets 60,558,000

Total shareholder’s equity 42,000,000

Question 29

The Statement of Financial Position of Golden Dawn Corporation on June 30,2020 is presented below:

Current Assets 195,000

Land 1,320,000

Building 660,000

Equipment 525,000

Total Assets 2,700,000

Liabilities 525,000

Ordinary Shares, P5 par 900,000

Share Premium 825,000

Retained Earnings 450,000

Total Equities 2,700,000

All the assets and liabilities of Golden Dawn assumed to approximate their fair values except for Land and Building. It is estimated
that the land has a fair value of P2,100,000 and the fair value of the building increased by P480,000. Enigma Corporation acquired
80% of Golden Dawn’s outstanding shares for P3,000,000. The non-controlling interest is measured at fair value.

Assuming the consideration paid excludes control premium of P138,000, and the fair value of non-controlling interest is P736,500,
how much is the goodwill/ (gain on acquisition) on the consolidated financial statement?

439,500
Assuming the consideration paid includes control premium of P222,000, how much is the goodwill/ (gain on acquisition) on the
consolidated financial statement?

259,500

Question 30

On January 1. 2018, Entity A acquired 30,000 out of 100,000 outstanding ordinary shares of Entity B for P90,000 or 30% interest.
For the six months ended June 30, 2018, Entity B reported net income of P40,000

On July 1, 2018, Entity A acquired additional 60,000 ordinary shares of Entity B or 60% interest at a price of P4 per share or total
cost of P240,000. Entity A paid P20,000 acquisition related costs and P10,000 indirect costs of business combination

The acquisition price per share of the additional shares clearly reflected the fair value of the existing interest of Entity A in Entity B. It
is the policy of Entity A to initially measure the noncontrolling interest in net assets of the acquiree at fair value. The fair value of the
noncontrolling interest in net assets of the acquiree is reliably measured at P50,000.

At the acquisition date, the net assets of Entity B were reported at P400,000. An asset of Entity B was overvalued by P50,000 while
one liability was overvalued by P30,000.

What is the gain on remeasurement of the existing Investment in Entity B as a result of step acquisition?

18,000

What is the goodwill or gain on bargain purchase as a result of the business combination?

30,000 goodwill

Question 31

Entity A acquired the net assets of Entity B by issuing 10,000 ordinary shares with par value of P10 and bonds payable with face
amount of P500,000. The bonds are classified as financial liability at amortized cost.

At the time of acquisition, the ordinary shares are publicly quoted at P20 per share. On the other hand, the bonds payable is trading
at 110.

Entity A paid P10,000 share issuance costs and P20,000 bond issue costs. Entity A also paid P40,000 acquisition related costs and
P30,000 indirect costs of business combination.
Before the date of acquisition, Entity A and Entity B reported the following data:

Entity A Entity B

Current assets 1,000,000 500,000

Noncurrent assets 2,000,000 1,000,000

Current liabilities 200,000 400,000

Noncurrent liabilities 300,000 500,000

Ordinary shares 500,000 200,000

Share premium 1,200,000 300,000

Retained earnings 800,000 100,000

At the time of acquisition, the current assets of Entity A have fair value of P1,200,000 while the noncurrent assets of Entity B have
fair value of P1,300,000. On the same date, the current liabilities of Entity B have fair value of P600,000 while the noncurrent
liabilities of Entity A have fair value of P500,000.

What is the goodwill or gain on bargain purchase arising from business combination?

50,000 goodwill

What is Entity A's amount of total liabilities after the business combination?

2,130,000

Question 32

IDEAL Corporation is a company involved in manufacturing mining equipment. At the beginning of the year, the board of directors of
the said company has decided to enter into a business combination with SUPERIOR Corporation and BRIGHT Corporation, top
suppliers of materials in the mining industry which they use in production. The said acquisition is expected to result in producing
higher quality mining equipment with lower total cost. The deal was closed on February 28,2020 and the following information was
gathered from the books of the entities:

IDEAL SUPERIOR BRIGHT

Current Assets 8,250,000 2,340,000 1,560,000


Noncurrent Assets 18,750,000 15,300,000 10,200,000

Total Assets 27,000,000 17,640,000 11,760,000

Liabilities 1,950,000 1,260,000 840,000

Ordinary Shares P100 par 16,491,000 10,681,200 7,120,800

Share premium 1,059,000 1,018,000 679,200

Retained Earnings 7,500,000 4,680,000 3,120,000

Total Equities 27,000,000 17,640,000 11,760,000

IDEAL, who has the legal and economic entity, will issue 135,000 of its ordinary shares in exchange for the acquisition of
SUPERIOR and 67,200 of its ordinary shares in exchange for the acquisition of BRIGHT.

The fair value of IDEAL’s shares is P150. In addition, the following adjustments should be made to the current assets Superior and
Bright which has a fair value of P2,700,000 and P1,380,000, respectively. The noncurrent assets have a fair value of P12,900,000
and P11,850,000 for Superior and Bright, respectively.

Compute for the following balances in the books of the surviving company on the date of acquisition.

Stockholder’s Equity

57,690,000

Assets

61,740,000

Question 33

The balance sheet of Piedmont Enterprises and Skelton Company at December 31, 2018 are summarized as follows:

Piedmont Skelton

Assets P5,000,000 P2,000,000

Liabilities P1,500,000 P500,000

Capital stock, P40 par 2,500,000


Capital stock, P25 1,000,000

Retained earnings 1,000,000 500,000

At the date of acquisition, Skelton's assets are understated while its liabilities are fairly valued.

On January 1, 2019, Piedmont purchased 80% of Skelton Company's outstanding shares for P2,000,000 when the fair value of
Skelton's net assets was P2,200,000. Piedmont issued 10,000 previously unissued shares in consideration of the acquisition.
Piedmont is to assign an amount to the non-controlling interests at the date of acquisition based on the total fair value of Skelton's
outstanding shares.

How much is the consolidated assets at the date of acquisition?

P8,000,000

How much is. the consolidated liability at the date of acquisition?

A. P2,000,000

How much is the stockholders' equity in the consolidated balance at January 1, 2019?

P6,000,000

Assume the amount assigned to the non-controlling interest at the date of acquisition is based on the total fair value of. identifiable
net assets at that date, calculate the amount of goodwill recognized at January 1, 2019.

P240,000

Question 34

Bullen Inc. acquired assets and liabilities of Vicker Inc. on January 1, 2021. The book value and fair value of Vicker's accounts on
that date (prior to creating the combination) follow, along with the book value of Bullen's accounts:

Bullen Vicker Vicker

Item Book Value Book Value Fair Value


Retained Earnings1/1/21 P 160,000 P240,000

Cash and receivables 170,000 70,000 P70,000

Inventory 230,000 170,000 210,000

Land 280,000 220,000 240,000

Buildings (net) 480,000 240,000 270,000

Equipment (net) 120,000 90,000 90,000

Liabilities 650,000 430,000 420,000

Common Stock 360,000 80,000

Additional paid-in capital 20,000 40,000

Assume that Bullen issued 12,000 shares of common stock with a P5 par value and a P47 fair value to obtain all of Vicker's
outstanding stock. In this transaction how much goodwill should be recognized?

P60,000

Assume that Bullen issued 12,000 shares of common stock with a P5 par value and a P42 fair value for all of the outstanding
shares of Vicker. What will be the Additional Paid-In Capital and Retained Earnings after the combination?

P464,000 and P160,000

Assume that Bullen issued preferred stock with a par value of P240,000 and a fair value of P500,000 for all of the net assets of
Vicker in a business combination. What will be the balance in the Inventory and Land accounts after the business combination?

P440,000 and P520,000

Assume that Bullen paid a total of P480,000 in cash for all of the shares of Vicker. In addition, Bullen paid P35,000 to a group of
attorneys for their work in arranging the combination to be accounted for as an acquisition. What will be the balance in goodwill?

A. P 0

Question 35
Great Company has gained control over the operations of Superb Corporation by acquiring 85% of its outstanding capital stock for
P15,480,000. This amount includes a control premium of P180,000. Acquisition expenses, direct and indirect, amounted to,
P495,000 and P252,000 respectively.

GREAT SUPERB

Book Value Book Value

Cash
P21,249,000 768,000

Accounts Receivable 1,800,000 1,950,000

Inventories 3.300,000 2,160,000

Prepaid Expenses 891,000 750,000

Land 14,100,000 5,274,000

Building 9,360,000 3,348,000

Equipment 1.800,000 1,110,000

Goodwill - 1,800,000

Total Assets 52,500,000 17,160,000

Accounts Payable 4,050,000 1,518,000

Notes Payable 8,400,000 4,380,000

Ordinary shares, 50 par 20,400,000 4,800,000

Share premium 9,450,000 3,600,000

Retained earnings 10,200,000 2,862,000

Total equities P52,500,000 P17,160,000

The following were ascertained on the date of acquisition for the Acquired Corporation:

 The value of receivables and equipment has decreased by P150,000 and P84,000 respectively.

 The fair value of inventories are now P2,616,000 whereas the value of land and building have increased by P2,826,000 and
P642,000, respectively.

 There was an unrecorded accounts payable amounting to P162,000 and the fair value of notes is P4,428,000.
Compute for the following balances to be presented in the consolidated statement of financial position

on the date of business combination:

Total Assets

60,558,000

Total shareholder’s equity

42,000,000
Quiz 2
Chapter 2 Separate and Consolidated Financial Statements - Date of Acquisition
Chapter 3 Separate and Consolidated Financial Statements - Subsequent to Date Acquisition

Question 1

At the end of 2013, Poppilon Company’s shareholder’s equity includes ordinary share of P500,000 and sharepremium of P300,000.
Poppilon purchased a 70 percent interest in Shumberg Company on January 1, 2013, when the non-controlling interest in
Shumberg had a fair value of P90,000. No differential arose from the business combination. During 2013, Shumberg reports net
income of P20,000 and declares dividend of P5,000. The 2013 consolidated statement of financial position includes retained
earnings of P630,000 (controlling interest portion).

Determine the consolidated equity on December 31, 2013: P1,524,500

Question 2

Miguel Co. acquired all of the issued and outstanding shares of Gor Co. for a cash consideration of P 140,000. Miguel and Gor’s
condensed statement of financial position before acquisition show the following:

Miguel Gor

Current assets 9,240,000 4,480,000

Plant and equipment 4,480,000 2,520,000

Goodwill - 560,000

Total assets 13,720,000 7,560,000

Liabilities 5,040,000 3,360,000

Ordinary share capital 5,600,000 2,800,000

Share premium 1,680,000 840,000

Retained earnings 1,400,000 560,000


Total equities 13,720,000 7,560,000

How should the difference between the consideration paid by Miguel and the book value of Gor’s stocks beaccounted for in the
consolidation worksheet? (Show as increase (decrease) in Goodwill; Plant and equipment; and Gain on Bargain purchase

(P 560,000); P0 ; P 3,500,000

Question 3

Control is presumed to exist when the parent company owns directly or indirectly through subsidiaries

more than 50 % of the voting power of an entity.

at least 50 % of the ordinary share of an entity

at least 50 % of the preference and ordinary shares of an entity

more than 50% of the equity of an entity

Question 4

A Co. and B Co. combined and immediately ceased to exist upon the setting up of a new corporation. This iscalled:

combination

consolidation

merger

acquisition

Question 5

New Corporation, on January 1, 2012, acquired all the outstanding common shares of Old Co. by paying P200,000 cash. On this day,
the assets and liabilities of Old were as follows:
Receivables P 30,000

Inventory 90,000

Fixed assets 160,000

Goodwill 50,000

Liabilities 60,000

Per appraisal, the inventory and fixed assets, respectively, have fair values of P 75,000 and P 190,000. Howmuch positive goodwill
or gain on bargain purchase would be reported on the consolidated statement of financial position?

P 15,000 positive

P0

P 35,000 gain on bargain purchase

P 55,000 positive

Question 6

X owns 50% of Y voting shares. The board of directors consists of six members. X appoints three of them andY appoints the other
three. The casting of the vote at meetings always lies with the directors appointed by X. Does X have control over Y?

Yes, X holds 50% of the voting power and has the casting vote at the board meetings in the event there is nomajority decision.

No, control is equally split between X and Y.

No, X owns only 50% of the entity’s shares and therefore does not have control.

No, control can be exercised only through voting power, not through vote.

Question 7
On October 1, 2012, Phoenix Co. acquired 80 % of the outstanding ordinary shares of Sacramento Co. in a acquisition-type
business combination. Total cost of investment, excluding direct out-of-pocket costs, was P 480,000. The working paper
elimination entry for Phoenix and subsidiary on October 1, 2012 was as follows:

Ordinary share capital – Sacramento Co. P 100,000

Share premium – Sacramento Co. 120,000

Retained earnings – Sacramento Co. 180,000

Plant assets – Sacramento Co. 50,000

Goodwill (600,000 – 400,000 – 50,000) 150,000

Investment in Sacramento Co. P 480,000

Noncontrolling interests in net assets of Sasmuan Co. 120,000

If noncontrolling interest has been reflected at Sacramento’s shareholders’ equity on the date of acquisition, rather than at fair
value of net assets of the subsidiary, the noncontrolling interest to be presented in the consolidated statement of financial
position on October 1, 2012 would have been

P 80,000

Question 8

Entity P has 90% controlling interest in Entity S. On December 31, 2013, the carrying value of Entity S’s net assets in Entity P’s
consolidated financial statements is P100,000 and the carrying amount attributable to the noncontrolling interest’s in Entity S
(including the non-controlling interest’s share of accumulated other comprehensive income) is P10,000. On January 1, 2014,
Entity P sells 80% of the share in Entity S to a thirdparty for cash proceeds of P120,000. As a result of the sale, Entity P loses
control of Entity S but retains a 10% non-controlling interest in Entity S. The fair value of the retained interest on that date is
P12,000.

Determine the gain or loss on disposal (deconsolidation) should be:

P42,000 gain

Question 9
Which statement is incorrect concerning the preparation of consolidated financial statements?

When the reporting dates of the parent and a subsidiary are different, the differences shall be no more thansix months.

Intragroup balances, transactions, income and expenses shall be eliminated in full.

Consolidated financial statements shall be prepared using uniform accounting policies for like transactionsand other events in
similar circumstances.

The financial statements of the parent and its subsidiaries shall be consolidated on a line-by-line basis byadding together like
assets, liabilities, equity, income and expenses.

Question 10

Even when more than one-half of the voting rights is not acquired, control may be evidenced by power:

I to appoint to remove the majority of the members of the board of directors

II to govern the financial and operating policies of the other enterprise under s statue or an agreement

I and II

Question 11

The underlying principle behind the preparation of consolidated financial statements of two legally distinctcompanies is:

synergy

conservatism

legal entity

economic entity

Question 12

The Elko Co. acquired a 60% interest in the Piyaya Co. when Piyaya’s equity method comprised share capital of P100,000 and
retained earnings of P150,000.
Piyaya’s current statement of financial position shows share capital of P100,000, a revaluation reserve of P75,000 and retained
earnings of P300,000. Under PAS 27, Consolidated and Separate Financial Statements,what amount in respect of the non-controlling
interest should be included in Elko Co.’s consolidated statement of financial position?

P190,000

Question 13

(Reverse Acquisition) Ortigas, a private limited company, has arranged for Concrete Aggregates Co.(CAC), a public limited
company, to acquire it as a means of obtaining a stock exchange listing. CAC issues 15 million shares to acquire the whole of the
share capital of Ortigas (6 million shares). The fair value of the net assets of Ortigas and CAC are P30 million and P18 million
respectively. The fair value of each of the shares of Ortigas is P6 and the quoted market price of CAC’s shares is P2. The share
capital of CAC is 25 milliion shares after acquisition. Calculate the value of goodwill in the above acquisition.

P6 million

Question 14

Prito Company acquires Inkalot Inc. on January 1, 2013. The consideration exceeds the fair value of Inkalot’snet assets. On that
date, Prito has a building with a book value of P1,200,000 and a fair value of P1,500,000.Inkalot has a building with a book value
of P400,000 and fair value of P500,000.

What amounts in the Building account appear on Inkalot’s separate statement of financial position and on the consolidated
statement of financial position immediately after acquisition?

Push Down Accounting No Push Down accounting

A P400,000 and P1,600,000 P500,000 and P2,000,000

B P500,000 and P1,700,000 P400,000 and P1,700,000

C P400,000 and P1,700,000 P500,000 and P1,700,000

D P500,000 and P2,000,000 P400,000 and P2,000,000

Question 15

A parent company need not present consolidated financial statements under which of the requisites?
I the parent is itself a wholly-owned subsidiary, or is a partially-owned subsidiary of another entity and the other owners,
including those not otherwise entitled to vote, have been informed about, and do not object to, the parent not presenting
consolidated financial statements

II the parent did not file, nor is it in the process of filing, its financial statements wit a securities commission or other
regulatory organization for the purpose of issuing any class of instruments in a publicsecurities market

both I and II

Question 16

Ordinarily, consolidated statements of cash flows are prepared:

using a consolidation working paper that fits the format of the statement of cash flows.

from the separate parent company and subsidiary financial statements rather than from the consolidated income statements and
consolidated balance sheets.

from the consolidated income statements and consolidated balance sheets rather than from the separateparent company and
subsidiary financial statements.

Question 17

Working paper entries normally:

are posted to the general ledger accounts only when the financial statement approach is used

are posted to the general ledger accounts only when the trial balance approach is used

do not affect the general ledger accounts of any of the affiliated companies

are posted to the general ledger accounts for one or more of the affiliated companies.

Question 18

What is the proper treatment of gain on bargain purchase?


offset with other positive goodwill accounts

debited to Share premium

credited to income immediately

credited to income but only after reassessing whether there are errors in the calculation of the valuation ofassets and liabilities

Question 19

(PAS 27) Assume there have been no intercompany transactions. Which of the following is an incorrect statement
concerning the financial statements of a parent and its 60 % owned subsidiary?

In the parent’s separate financial statements, the investment in subsidiary should be either carried at cost, or accounted for in
accordance with PAS 39.

The noncontrolling interests in net assets would not be shown on the consolidated statement of financialposition.

Net income of the parent would be the same whether or not consolidated statements were prepared.

Consolidated financial statements would include 100 % of the assets and liabilities of the subsidiary.

Question 20

Parent Co. purchased an 80 % interest in Subsidiary Co. for P 230,000 on January 1, 2012, when Subsidiaryhad the following :

Assets Equities

Current assets P 100,000 Current liabilities P 50,000

Plant assets – net 200,000 Share Capital, P 10 par 100,000

Retained earnings 150,000


Total assets P 300,000 Total equities P 300,000

The applicable amortization of the excess of the price paid over the book value amounted to P 3,750.

The following trial balances of the two companies were prepared on December 31, 2012:

in Philippine Pesos

Parent Subsidiary

Current assets 80,000 130,000

Plant assets 400,000 200,000

Accu. dep’n. (106,000) (20,000)

Investment in Subsidiary Co. 230,000 -

Current liabilities ( 60,000) (40,000)

Common stock (par P 10) (300,000) (100,000)

Retained earnings, Jan. 1 (200,000) (150,000)

Sales (150,000) (100,000)

Costs and expenses 110,000 75,000


Dividend income (4,000)

Dividends declared 5,000

Net amount 0 0

The consolidated net income attributable to parent equity holders for 2012 is:

P 57,000

Question 21

Which of the following items will be eliminated in the consolidated statements?

I Goodwill arising from the acquisition

II Investment in the subsidiary

III Inter-company receivable and payables

IV Equity accounts of the subsidiary

V Non-controlling interest in the net assets of the subsidiary

II, III and IV

Question 22

According applicable PAS/PFRS, a subsidiary shall be excluded from consolidation when:

the Corporation Code so provides

control is intended to be temporary


its operations are different from parent activities

the shareholders are the same

Question 23

Double Co. purchased Simple Co. for P 450,000 on January 1, 2012. On that date, Simple’s identifiable net assets had a fair
value of P 390,000. The assets acquired in the purchase of Simple are considered to be aseparate reporting unit of Double. The
carrying value of Double’s investment at December 31, 2012 is P 500,000.

What amount of goodwill impairment, if any, should be recognized at December 31, 2012, if the value of the net assets (excluding
goodwill) at that date is P 440,000 and the fair value of the reporting unit is determinedto be P 485,000?

P 15,000

Question 24

PAS 27, on Consolidated Financial Statements states that “… consolidated statements are more meaningfulthan separate financial
statements and are usually necessary for a fair presentation when one of the companies in the group directly or indirectly has a:

controlling financial interest in the other companies

significant influence in the other companies

controlling technical interest in the other companies

Question 25

. In the separate balance sheets of the affiliates, which of the following would ordinarily be shown?

gain on bargain purchase

goodwill

investment in subsidiary

noncontrolling interest in net assets of a subsidiary


Question 26

On January 1, 2013, Willingwili Corporation pays P388,000 for a 60% ownership in Kapatid Corporation. Annual excess fair value
amortization of P15,000 results from the acquisition. On December 31, 2014, Kapatid reports revenues of P400,000 and
expenses of P300,000 and Willingwili reports revenues of P700,000 and expenses of P400,000. The parent figures contain no
income from the subsidiary. What is the consolidated income attributable to the controlling interest/ profit attributable to equity
holders of parent?

P351,000

Question 27

In the consolidated statement of financial position, which of the following would most likely be presented?

investment in subsidiary account

goodwill

inter-company receivable

inter-company payable

Question 28

On January 1, 2013, Head Inc, reports net assets of P480,000 although a building (with a 10 year life) having a book value of
P260,000 is now worth P300,000. Band Corporation pays P540,000 on that date for a 90 percent ownership interest in Head. On
December 31, 2015, Head reports a Building account of P182,000 and Band reports a Building account of P510,000. What is the
consolidated balance of the Building account?

P810,000

P780,000

P720,000

P724,000

Question 29
Which of the following is not a valid condition that will exempt an entity from preparing consolidated financialstatements?

The parent entity is a wholly owned subsidiary of another entity.

The ultimate parent entity produces consolidated financial statements available for public use that comply withPFRS.

The parent entity’s debt or equity capital is not traded in the stock exchange.

The parent entity is in the process of filing its financial statements with a securities commission for the purpose of issuing any
class of instruments in a public market.

Question 30

(PAS 27) A parent company need not present consolidated financial statements under which of the requisites?

I the parent’s debt or equity instruments are not traded in a public securities market

II the parent’s operations is compatible with the operations of the subsidiary

III the parent is itself a wholly-owned subsidiary, or is a partially-owned subsidiary of another entity and the other owners,
including those not otherwise entitled to vote, have been informed about, and do not object to, the parent not presenting
consolidated financial statements

I and III

Question 31

PAS 27 Par 4, As defined by accounting standards, control is the power to govern the
of an enterprise so as to obtain benefits from its activities.

I financial policies II operating policies III managerial policies

I and II
Question 32

Consolidated financial statements are typically prepared when a company has:

accounted for its investment in another company by the cost method

accounted for its investment in another company by the equity method

significant influence over the operating and financial policies of another company

the controlling financial interest in another company

Question 33

PAS 27, Par 10, A parent company need not present consolidated financial statements under which of therequisites?

I the parent’s debt or equity instruments are not traded in a public securities market

II the parent’s operations is compatible with the operations of the subsidiary

III the parent is itself a wholly-owned subsidiary, or is a partially-owned subsidiary of another entity and the other owners,
including those not otherwise entitled to vote, have been informed about, and do not object to, the parent not presenting
consolidated financial statements

Question 34

A parent company need not present consolidated financial statements under which of the requisites?

I the parent’s debt or equity instruments are not traded in a public securities market

II the parent is itself a wholly-owned subsidiary, or is a partially-owned subsidiary of another entity and the other owners,
including those not otherwise entitled to vote, have been informed about, and do not object to, the parent not presenting
consolidated financial statements

III. the ultimate or any intermediate parent of the parent produces consolidated financial statements availablefor use that comply
with PFRS
Question 35

When push-down accounting has been implemented:

The issuer and the combiner’s equity sections are merged.

Subsidiary records have been adjusted to reflect the market value increases resulting from the purchase by aparent company.

Any debt incurred by the parent in acquiring the subsidiary is recorded at its market value by the subsidiary.

The non-controlling interest in the subsidiary is shown on its own line in the equity section of the subsidiary only statement of
financial position.

Question 36

Noncontrolling interest in the net assets of a subsidiary should be presented

in the consolidated statement of financial position within equity

between liabilities and equity

in the separate balance of the parent within equity

as a liability

Question 37

On January 1, 2013, Potter Inc. reports net assets of P880,000 although a patent (with a 10 year life) having a book value of
P330,000 is now worth P400,000. Solum Corporation pays P840,000 on that date for an 80 percent ownership of Solum. On
December 31, 2014, Potter reports total expenses of P621,000 while Solum reports expenses of P714,000. What is the
consolidated total expense balance on December 31, 2014?

P1,349,000

P1,197,800

P1,342,000

P1,335,000
Question 38

On January 1, 2012, Pie Co. pays P 64,000 cash and also issues 18,000 shares of P 10 par common stock with a market value of
P 216,000 for all the outstanding shares of Cutie Co. In addition, Pie pays P 30,000 forregistering and issuing the equity shares
and P 70,000 for the other direct costs of the acquisition.

Summary balance sheet information for the companies immediately before the transaction is as follows:

Pie Cutie Cutie

Book Value Book Value Fair value

Cash 350,000 40,000 40,000

Inventories 120,000 80,000 100,000

Other noncurrent assets 30,000 20,000 20,000

Plant assets – net 260,000 180,000 280,000

Total assets 760,000 320,000 440,000

Current liabilities 160,000 30,000 30,000

Other liabilities 80,000 50,000 40,000

Common stock P 10 par 420,000 200,000


Retained earnings 100,000 40,000

Total equities 760,000 320,000

===============================================

1. On January 1, 2012, the Share premium amounted to:

(a) P 6,000 (b) P 30,000 (c) P 36,000 (d) P 100,000

2. How much is the Goodwill or gain on bargain purchase?

(a) P 0 (c) P 90,000 Goodwill

(b) P 90,000 Gain on bargain purchase (d) P 80,000 Gain on bargain purchase

Question 39

On January 1, 2013, Parentis Company purchased 80%, of the ordinary shares of Sonsona Company for P316,000. On this
date, Sonsona Company had ordinary share, share premium and retained earnings of P40,000, P120,000 and P190,000
respectively. Parentis Company’s ordinary share amounted to P500,000and retained earnings of P200,000.

On January 1, 2013, the only tangible assets of Sonsona that were undervalued were inventory and building. Inventory, for which FIFO
is used, was worth P5,000 more than cost. The inventory was sold in 2013. Building,which was worth P15,000 more than book value,
has remaining life of 8 years, and straight-line depreciation
is used. Any remaining excess is full goodwill with an impairment for 2013 amounting to P3,000. Sonsona Company reported net
income of P50,000 and paid dividends of P10,000 in 2013 while the parent’s reportednet income amounted to P100,000 and paid
dividends of P20,000.
1. Determine the consolidated net income attributable to controlling interest/ profit attributable to equityholders of parent:

A. P142,000

B. P132,125

C. P126,500

D. P124,100

2. Compute the equity holders of Parentis – Retained earnings / Controlling interest in the ConsolidatedRetained Earnings

A. P200,000

B. P324,100

C. P304,100

D. P342,125

Question 40

On October 1, 2013, separate statements of Goldie Co. and Bata Co. appear below:

Goldie Bata

Cash 59,700 7,500

Accounts receivable 136,000 23,900


Inventories 57,300 9,250

Plant and equipment 286,300 13,600

Total assets 539,300 54,250

Liabilities 123,800 11,900

Share Capital 100,000 10,000

Share premium 25,000 -

Retained earnings 290,500 32,350

Total equities 539,300 54,250

Goldie acquired an 80 % interest in Bata Co. On the acquisition date, October 1, 2013, the fair values of Bata’s assets were
properly reflected in its accounts. P 40,000 was paid for this acquisition. The transaction was treated as a purchase. Goldie
recognized the non-controlling interest in Bata Co. at its proportionate share of Bata’s identifiable net assets.

1. In the preparation of a consolidated statement of financial position, the elimination entry as to goodwill in the consolidated
working paper will be:
A. a credit to the investment account by P6,120

B. a credit to the investment account by P7,670

C. a debit to the investment account by P3,178

D. a credit to the plant and equipment account by P6,120

2. The working paper elimination entry pertaining to capital stocks and retained earnings of the subsidiarycompany is

(a) Share Capital – Bata 10,000

Retained earnings – Bata 32,350

Investment in Bata 33,880

Non-controlling interest in net assets of subsidiary 8,470

(b) Share Capital – Bata 10,000

Retained earnings – Bata 32,350

Investment in Bata 42,350

(c) Share Capital – Bata 10,000

Retained earnings – Bata 32,350


Goodwill 5,300

Investment in Bata 40,000

Non-controlling interest in net assets of subsidiary 7,650

d. Share Capital – Bata 100,000

Retained earnings – Bata 290,500

Goodwill 7,650

Investment in Bata 40,000

Unamortized excess 358,150

3. The non-controlling interest in the consolidated statement of financial position will be:

a. P8,470

b. P5,300

c. P7,650

d. P10,850

e. P6,470

Question 41
The statement of financial position of P Co. and S Co., affiliates, on the date of acquisition are as follows:

P Co. S Co.

Cash P 20,000 P 8,000

Accounts receivable 40,000 32,000

Inventory 50,000 20,000

Building – net 200,000 100,000

Equipment – net 80,000 50,000

Investment in S Co. 110,000

Total assets P 500,000 P 210,000

Accounts payable P 280,000 P 110,000

Ordinary share capital:

P Co. 100,000

S Co. 50,000

Share premium:
P Co. 80,000

S Co. 30,000

Retained earnings:

P Co. 40,000

S Co. 20,000

Total equities P 500,000 P 210,000

P Co. acquired 100 % of the voting stocks of S Co. for P 110,000 cash on December 31, 2012.

1. The necessary elimination entry should be

(a) Ordinary share capital – S Co. P 50,000

Share premium – S Co. 30,000

Retained earnings – S Co. 20,000

Investment in S Co. P 100,000

(b) Ordinary share capital – S Co. P 50,000

Share premium – S Co. 30,000


Retained earnings – S Co. 20,000

Goodwill 10,000

Investment in S Co. P 110,000

(c) Ordinary share capital – S Co. P 50,000

Share premium – S Co. 30,000

Retained earnings – S Co. 20,000

Goodwill 10,000

Cash P 110,000

(d) no entry

2. In the consolidated statement of financial position as of December 31, 2012, how much would be shown astotal assets?

(a) P 710,000

(b) P 500,000

(c) P 510,000

(d) P 610,000
3. In the statement of financial position of the parent as of December 31, 2012, how much would be shown asInvestment in S Co.?

(a) P 110,000 (b) P 100,000 (c) P 120,000 (d) P 0

Question 42

On January 3, 2013, Montiel Company acquired 80 percent of Donaire Corporation’s ordinary share for P344,000 in cash. At the
acquisition date, the book values and fair values of Donaire’s assets and liabilities were equal, and the fair value of the non
controlling interest was equal to 20% of the total book value of Donaire. The shareholders’ equity accounts of the two companies
at the acquisition date
are: Montiel Donaire

Ordinary share, P 5 par value P 500,000 P 200,000

Share premium 300,000 80,000

Retained earnings 350,000 150,000

Total Shareholder’s Equity P1,150,000 P430,000

Non-controlling interest was assigned income of P11,000 in Montiel’s consolidated income statement for 2013.

1. What will be the amount of net income reported by Donaire Corporation in 2013?

a. P44,000

b. P55,000

c. P66,000

d. P36,000

2. What amount will be assigned to the non-controlling interest on January 3, 2013 in the consolidated statement of
financial position?
a. P86,000

b. P44,000

c. P68,800

d. P50,000

3. What will be the total stockholders’ equity in the consolidated statement of financial position as of January 3,2013?

a. P1,580,000

b. P1,064,000

c. P1,150,000

d. P1,236,000

Question 43

Wilkins, Inc acquires all of the outstanding stock of Premier Corp on January 1, 2013. At that date, Premier owns only three
assets and has no liabilities:

Book Value Fair Value

Inventory P 40,000 P 50,000

Equipment (10 year life) 80,000 75,000

Building (20 year life) 200,000 300,000


1. If Wilkins pays P450,000 in cash for Premier, what amount would be represented as the subsidiary’s Building in a
consolidation at December 31, 2015, assuming the book value at that date is still P200,000?

A. P200,000

B. P255,000

C. P285,000

D. P300,000

2. If Wilkins pays P400,000 in cash for Premier, what amount would be represented as the subsidiary’s Building in a
consolidation at December 31, 2015, assuming the book value at that date is still P200,000.

A. P200,000

B. P255,000

C. P285,000

D. P300,000

3. If Wilkins pays P450,000 in cash for Premier, what amount would be represented as the subsidiary’s Equipment in a
consolidation at December 31, 2015, assuming the book value at that date is still P80,000.

A. P70,000

B. P73,500

C. P76,500

D. P80,000
4. If Wilkins pays P450,000 in cash for Premier, what allocation should be assigned as the subsidiary’s Equipment in a
consolidation at December 31, 2015, assuming the book value at that date is still P80,000.

A. P3,500

B. P5,000

C. P75,000

D. P80,000

Question 44

Maglaya Company acquired 90 percent of Hangganan Company on January 1, 2013, for P234,000 cash. Hangganan’s
shareholders’ equity consisted of ordinary share capital of P160,000 and retained earnings of P80,000. An analysis of
Hangganan’s net assets revealed the following:

Any excess consideration transferred over fair value is attributable to an unamortized patent with a useful lifeof 5 years.

Book value Fair value

Building (10 year life) P10,000 P 8,000

Equipment (4 year life) 14,000 18,000

Land 5,000 12,000

1. In consolidation at January 1, 2013, what adjustment is necessary for Hangganan’s Building account?

A. P2,000 increase C. P1,800 increase

B. P2,000 decrease D. P1,800 decrease


2. In consolidation at December 31, 2013, what adjustment is necessary for Hangganan’s Buildings account?

A. P1,620 increase C. P1,800 increase

B. P1,620 decrease D. P1,800 decrease

3. In consolidation at January 1, 2013, what adjustment is necessary for Hangganan’s Land account?

A. P7,000 increase C. P6,300 increase

B. P7,000 decrease D. P6,300 decrease

4. In consolidation at December 31, 2013, what adjustment is necessary for Hangganan’s Land account?

A. P0

B. P7,000 decrease

C. P7,000 increase

D. P6,300 decrease

Question 45

Bola Company acquires 80% of Dumaghoy Company for P500,000 on January 1, 2013. Dumaghoy reported ordinary share of
P300,000 and retained earnings of P200,000 on that date. Equipment was undervalued by P30,000 and buildings were
undervalued by P40,000, each having a 10 year remaining life. Any excess consideration transferred over fair value was attributed
to goodwill with an indefinite life. Based on the an annual review of goodwill has not been impaired.

Dumaghoy earn income and pays dividends as follows:

2013 2014 2015


Net income P 100,000 P 120,000 P 130,000

Dividends 40,000 50,000 60,000

Assume the initial value method (or cost method) as applied.

1. Compute Bola’s investment in Dumaghoy at December 31, 2013:

A. P500,000 B. P542,400 C. P574,400 D. P625,000

2. Compute Bola’s investment in Dumaghoy at December 31, 2015:

A. P676,000 B. P625,000 C. P592,400 D. P500,000

3. How much does Bola report as Income from Dumaghoy/ Dividend income for the year ended December 31,2015?

A. P48,000 B. P50,400 C. P56,000 D. P98,400

4. Compute the non-controlling interest in the net income of Dumaghoy at December 31, 2014

A. P14,000 B. P18,400 C. P22,600 D. P24,000

5. Compute the non-controlling interest of Dumahoy using full-goodwill method at December 31, 2015:

A. P80,000 B. P107,800 C. P140,000 D. P160,800

Question 46

On January 1, 2012, Parent Co. acquired 90 % of Subsidiary Co. in exchange for 5,400 shares of P 10 par ordinary share having
a market value of P 120,600. Parent and Subsidiary condensed statement of financial position (before combination) were as
follows:

(in Philippine Pesos)


Parent Subsidiary

Assets

Cash 30,900 37,400

Accounts receivable-net 34,200 9,100

Inventories 22,900 16,100

Equipment – net 179,000 40,000

Patents - 10,000

Total assets 267,000 112,600

Equities

Accounts payable 4,000 6,600

Bonds payable, 10 % 100,000 -

Share Capital, P 10 par 100,000 50,000

Share premium 15,000 15,000

Retained earnings 48,000 41,000

Total equities 267,000 112,600

At the date of acquisition, all assets and liabilities of Subsidiary have book value approximately equal to their respective market
values except the following as determined by appraisal as follows:
Inventories (FIFO method) 17,100

Equipment (net – remaining life – 4 years) 48,000

Patents (remaining life 10 years) 13,000

1. How much is the amount of goodwill on January 1, 2012?

(a) P 12,000 (b) P 16,000 (c) P 14,400 (d) P 25,200

2. How much is the non-controlling interest in the subsidiary at fair values on January 1, 2012?

(a) P 10,600 (b) P 11,200 (c) P 11,800 (d) P 13,400

3. In addition, assuming that on December 31, 2012, the following results of operations were given:

Dividends paid Net income

Parent Co. P 15,000 P 30,200

Subsidiary Co. 4,000 9,400

Assume also that the entity prepares separate financial statements is using the cost method. How much is the
investment in subsidiary co. balance on December 31, 2012?

(a) P 0 (b) P 120,600 (c) P 122,160 (d) P 125,460

4. Compute the consolidated retained earnings on January 1, 2012.

(a) P 41,000 (b) P 48,000 (c) P 60,600 (d) P 89,000


5. Compute the net income attributable to non-controlling interest on December 31, 2012.

(a) P 940 (b) P 540 (c) P 610 (d) P 310

6. Compute the non-controlling interest at the end of 2012.

(a) P 13,940 (b) P 13,610 (c) P 12,010 (d) P 14,710

7. Compute the consolidated net income attributable to parent equity holders for 2012.

(a) P 26,600 (b) P 31,760 (c) P 32,090 (d) P 39,390 (e) 44,100

8. Compute the consolidated retained earnings on December 31, 2012.

(a) P 63,200 (b) P 74,990 (c) P 65,090 (d) P 68,600

9. Compute the consolidated shareholders’ equity on December 31, 2012.

(a) P 300,690 (b) P 314,300 (c) P 312,700 (d) P

Intercompany Transactions Part 1 Quiz

Question 1

Sailing Company owns 100% of the capital stock of both Twill Corp. and Webb Corp. Twill purchases merchandise inventory from
Webb at 140% of Webb's cost. During 2012, merchandise that cost Webb P40,000 was sold to Twill. Twill sold all of this
merchandise to unrelated customers for P81,200 during 2012. In preparing combined financial statements for 2012 Sailing's
bookkeeper disregarded the common ownership of Twill and Webb. By what amount was unadjusted revenue overstated in the
combined income statements for 2012 and the amount that should be eliminated from cost of goods sold in the combined income
statement for 2012?

Overstated Unadjusted Cost of goods sold


Revenue to be eliminated

A.P16,000 P16,000

B. 40,000 40,000

C. 56,000 56,000

D. 81,200 56,000

Question 2
On January 1, 2013, Par Company purchased 80% of the outstanding shares of Sub Company by paying P340,000, the Sub
Company's common stock and retained earnings on this date amounted to P150,000 and P230,000 respectively. Also on this date,
an equipment is undervalued by P20,000 with a remaining life of 10 years.

On January 1, 2015, Sub Company had P150,000 of capital stock and P300,000 or retained earnings. Also on the same date, Par
Company had P1,000,000 of capital stock and P700,000 of retained earnings.

During the year, Par Company sold merchandise to Sub for P60,000 and in turn, purchased P40,000 from Sub Company. Inter-
company sales of merchandise were made at the following gross profit rates:

Sales made by parent 25% based on cost

Sales made by subsidiary 20% based on sales


On December 31, 2015, 30% of all inter-company sales remain in the ending inventory of the purchasing affiliate.

The beginning inventory of Par Company includes P2,500 worth of merchandise acquired from Sub Company on which Sub
Company reported a profit of P1,000. While, the beginning inventory of Sub also includes P3,000 of merchandise acquired from Par
Company at 35% mark-up.

Using cost method the following selected results of operations for 2015 were as follows:

Par Company Sub Company

Dividends paid P 60,000 P10,000

Net income from own operations P100,000 P30,000

Add: Dividend income 8,000

Net income P108,000 P30,000

The balance of Investment in Sub Company as of December 31, 2015 should be:

A. P354,600 C. P350,330

B. P351,960 D. P340,000

the parent's portion of consolidated (for controlling interest / equity holders of parent) retained earnings on December 31,
2015:

A. P700,000 C. P753,600

B. P752,000 D. P809,680

The stockholders' equity of subsidiary on December 31, 2015:

A. P450,000 C. P481,600

B. P470,000 D. P484,000

Question 3
The Non-controlling Interest (in Net Assets) on December 31, 2015 using proportionate basis (or partial goodwill approach) should
be:

P97,120
P96,920
P96,320
P73,520

Question 4
The Non-controlling interest (in Net Assets) on December 31, 2015 using full fair value basis (or full-goodwill approach) should be:

P96,320
P73,520
P96,920
P101,320

Question 5
Use the following information for the next four questions:

On September 1, 20x1, Pig Co. acquired 75% interest in Piglet Co. At this time, Piglet's net identifiable assets have a carrying
amount of ₱720,000 which approximates fair value. During the last month of the year, Piglet sold goods to Pig for ₱324,000. Piglet
had marked up these goods by 50% on cost. Onethird of these goods remain unsold at year-end. The group assessed that there is
no impairment loss on goodwill for the current year.

The individual statements of profit or loss of the entities for the year ended December 31, 20x1 are shown below:

Pig Co. Piglet Co.

Revenue 4,000,000 2,880,000

Cost of Sales (1,600,000) (1,200,000)

Gross Profit 2,400,000 1,680,000

Distribution Costs (800,000)

Administrative Costs (320,000) (180,000)

Profit before tax 1,280,000 1,100,000


Income Tax Expense (384,000) (380,000)

Profit after Tax 896,000 720,000

All of Piglet’s income and expenses (including profit from intercompany sale) were earned and incurred evenly during the year.

How much is the consolidated sales?

A. 6,556,000

B. 4,852,000

C. 4,786,000

D. 4,636,000

How much is the consolidated cost of sales?

A. 1,712,000

B. 2,530,000

C. 1,730,000

D. 1,876,000

How much is the consolidated profit?

A. 1,100,000

B. 1,580,000

C. 1,360,000

D. 1,420,000

How much is the profit attributable to owners of the parent and NCI, respectively?

Owners of Parent NCI

A 1,040,000 60,000

B 1,049,000 51,000

C 1,036,000 544,000

D 1,049,000 311,000

Question 6
Bear Co. owns 75% of Cub Co.’s ordinary shares. Cub Co. has 12%, ₱400,000 outstanding cumulative preference shares, none of
which are held by Bear Co. The carrying amount of Cub’s net identifiable assets at acquisition date approximates fair value.

Bear and Cub reported individual profits of ₱936,000 and ₱700,000, respectively, for the year ended December 31, 20x1. Neither
company declared dividends. There are 3-year dividends in arrears on the outstanding cumulative preference shares of Cub Co. It
was assessed that goodwill is not impaired.

How much is the profit attributable to owners of the parent and NCI, respectively?

Owners of Parent NCI

A 1,425,000 163,000

B 1,377,000 163,000

C 1,377,000 211,000

D 1,425,000 211,000

Question 7
The Maroons Company holds a 70% interest in the Haena Company. At the current year end Maroons holds inventory purchased
from Haena for P270,000 at cost plus 20%. The group’s consolidated statement of financial position has been drafted without any
adjustments in relation to this holding of inventory.

What adjustments should be made to the draft consolidated statement of financial position figures for non-controlling interest and
retained earnings?
Non-controlling interest Retained earnings

A. No change Reduce by P45,000

B. No change Reduce by P54,000

C. Reduce by P16,200 Reduce by P37,800

D. Reduce by P13,500 Reduce by 31,500

Question 8
Rosas Corp. acquired a 70% interest in Camia Co. in 2011. For the year ended December 31, 2011 and 2012, Camia Co. reported
net income of P160,000 and P 180,000, respectively. During 2011, Camia sold merchandise to Rosas Corp. for P20,000 at a profit
of P4,000. The merchandise was later resold by Rosas Corp. to outsider for P30,000 during 2012. For consolidation purposes, what
is the non-controlling interest's share of Camia's net income for 2011 and 2012, respectively?

2011 2012

A. P46,800 P55,200

B. P48,000 P54,000

C. P49,000 P52,800

D. P53,200 P50,000

Question 9
On July 1, 2020, Entity A acquired 70% of outstanding shares of Entity B in exchange for P4,000,000 cash and 100,000 preferred
shares with par value of P100 and quoted price of P200. On the acquisition date, the net assets of Entity B has book value of
P39,000,000 but a building with remaining useful life of 10 years is overstated by P4,000,000. On October 1, 2020, Entity B sold an
equipment to Entity A at a gain of P200,000. On the date of sale, the equipment has remaining life of 5 years. For the year ended
December 31, 2020, Entity B reported net income of P900,000 and declared dividends of P100,000 on December 30, 2020. On the
other hand, Entity A reported net income of P2M on its separate income statement and declared dividends of P1,000,000. The
retained earnings of Entity A on December 31, 2019 is P10,000,000.

What is Entity A's consolidated net income year ended December 31, 2020?

A. P2,830,000

B. P2,130,000

C. P2,990,000

D. P3,120,000

What is Entity A's consolidated net income attributable to parent's shareholders for the year ended December 31, 2020?

A. P2,822,000

B. P2,648,000

C. P2,448,000

D. P2,288,000

What is noncontrolling interest in net assets as of December 31, 2020?

A. P10,638,000

B. P10,112,000

C. P11,092,000

D. P9,112,000

What is the consolidated retained earnings as of December 31, 2020?

A. P10,120,000

B. P11,822,000

C. P12,130,000

D. P10,480,000

Question 10
On January 1, 2021, Entity A acquired (80%) of outstanding ordinary shares of Entity B. with a gain on bargain purchase amounting
to P1,000,000. The following additional data are provided:

On January 1, 2021, Entity A sold a black equipment to Entity B with cost of P1,000,000 and accumulated depreciation of P400,000
at a selling price of P900,000. The black equipment has original life of 5 years with no residual value.
On July 1, 2022, Entity B sold a white equipment to Entity A with cost of P500,000 and accumulated depreciation of P300,000 at a
selling price of P150,000. The white equipment has original life of 10 years with no residual value.
On year 2022, Entity A reported net income of P5,000,000 and declared dividends of P2,000,000 while Entity B reported net income
of P1,000,000 and declared dividends of P500,000.

What is the consolidated net income attributable to parent's shareholders for the year ended December 31, 2022?

A. P5,326,250

B. P5,535,000

C. P6,422,500

D. P6,622,500

What is the noncontrolling interest in net income for the year ended December 31, 2022?

A. P208,750

B. P219,000

C. P205,000

D. P225,000

What is the consolidated depreciation expense of the equipment for the year ended December 31, 2022?

A. P250,000

B. P225,000

C. P125,000

D. P112,500

What is the consolidated book value of the equipment on December 31, 2022?

A. P375,000

B. P412,500

C. P350,000

D. P425,000

Question 11
Onyx Co., a 100% owned subsidiary of OPAL Corp. began operation on January 1, 20x5. The following is from their 20x5 income
statements:

OPAL ONYX

Sales to Onyx P100,000 P0

Sales to others 400,000 300,000

Total Sales P500,000 P300,000

Cost of goods sold:

Acquired from Opal P0 P80,000

Acquired from others 350,000 190,000

Total cost of goods sold P350,000 P270,000

Gross profit P150,000 P30,000

Depreciation 40,000 10,000

Other expenses 52,000 15,000

Operating Income P58,000 P5,000

Gain on sale of equipment 12,000 0


Income before tax P70,000 P5,000

Intercompany sales carry the same markup as sales to others; equipment sold by OPAL to Onyx in January 1, 20x5 for P36,000, is
being depreciated over four years by straight-line method.

In OPAL's December 31, 20x5 consolidating worksheet, what amount of intercompany profit should be eliminated from
Onyx's inventory?

A. P6,000

B. P10,000

C. P20,000

D. P30,000

In OPAL's 20x5 consolidated income statement, what amount should be reported as depreciation expense?

A. P41,000

B. P44,000

C. P47,000

D. P50,000

Question 12
HAZEL Co. acquired its 60% interest in Hyper Co. four years ago for P200,000, and has accounted for its investment by the equity
method. At the time of the acquisition, the purchase premium has been identified as follows:

Inventory = P 8,000 (sold in year following purchase)

Equipment = 30,000 (15-year life)

Goodwill = 12,000 (40-year benefit)

For the first three years after acquisition, Hyper Co. had reported total earnings of P90,000 and paid total dividends of P50,000. In
20x6, the current year, the parent company earned P60,000 and paid P40,000 for dividends, while the subsidiary earned P30,000
and paid P20,000 for dividends.

The consolidated net income for 20x6 was


A. P67,700

B. P75,700

C. P78,000

D. P90,000

On HAZEL Company's books, the investment's carrying value at the end of the current year would be
A. P200,000

B. P212,800

C. P222,000

D. P230,000

Question 13
The following working paper elimination entry appears in the consolidation worksheet for TRIUMPH, Inc. and its subsidiary, UNITED
Company, to eliminate unrealized intercompany profit on machinery and related depreciation (ignoring tax effect)

Retained earnings - Units (450,000x 0.9 405,000

Minority interest in Net Assets of United (450,000 x 0.1) 45,000

Accum. depreciation - Triumph 300,000

Machinery - Triumph 600,000


Depreciation Triumph (St. P600,000/4) 150,000

How many years have passed since TRIUMPH acquired the machinery from its subsidiary, UNITED?

2 Years
½ Year
3 Years
1 Year
Question 14
The Snipes Company owns 65% of The Genie Company. On the last day of the accounting period Genie sold to Snipes a non-
current asset for P200,000. The asset originally cost p500,000 and at the end of the reporting period its carrying amount in genie's
books was P160,000. The group's consolidated statement of financial position has been drafted without any adjustments in relation
to this non-current asset.

What adjustments should be made to the consolidated statement of financial position figures for non-current assets and
retained earnings?

Non-current assets Retained earnings

A. Increase by P300,000 Increase by P195,000

B. Reduce by P40,000 Reduce by P26,000

C. Reduce by P40,000 Reduce by P40,000

D. Increase by P300,000 Increase by P300,000

Question 15
The Virgil Company owns 65% of The Migu Company. On December 31, 2012, the last day of the accounting period, Virgil sold to
Migu a noncurrent asset for P1,000. The asset's original cost was p2,500 and on December 31, 2012 its carrying amount in Virgil's
books was P800. The group's consolidated statement of financial position has been drafted without any adjustments in relation to
this non-current asset.

What adjustments should be made to the consolidated statement of financial position figures for non-current assets and non-
controlling interest?

Non-current assets Non-controlling interest

A. Increase by P1,500 Increase by P525

B. Reduce by P200 No change

C. Reduce by P200 Reduce by P70

D. Increase by P1,500 No change

Question 16
The Roel Company acquired equipment on January 1, 2009 at a cost of P800,000, depreciating it over 8 years with a nil residual
value. On January 1, 2012. The Muldon Company acquired 100% of Roel and estimated the fair value of the equipment at
P460,000, with a remaining life of 5 years. This fair value was not incorporated into Roel's books and the depreciation expense
continued to be calculated by reference to original cost.

What adjustments should be made to the depreciation expense for the year and the statement of financial position carrying amount
in preparing the consolidated financial statements for the year ended December 31, 2013?

Depreciation expense Carrying amount

A. Increase by P8,000 Increase by P24,000

B. Increase by P8,000 Decrease by P24,000

C. Decrease by P8,000 Increase by P24,000

D. Decrease by P8,00 Decrease by P24,000

Question 17
As January 1, 2012, Johnson Corporation sold equipment with a three-year remaining useful life and a book value of P10,000 to its
70% owned subsidiary for a price of P11,500. In the consolidation working papers for the year ended December 31, 2012, the
elimination entry concerning this transaction will include:

A debit to gain on equipment sale for P1,000.


A credit to depreciation expense for P1,500.
A debit to equipment for P1,500.
A debit to gain on equipment for P1,500.

Question 18
On January 1, 2011, Pure Company purchased 80 percent of the outstanding shares of Sure Company at a cost of P1.000,000. On
that date, Sure Company had P400,000 of capital stock and P600,000 of retained earnings.

On July 1, 2011, Sure Company sold an equipment with a book value of P60,000 to Pure Company for P80,000.

For 2011 and 2012, the results of their operations are:

2011 2012

Pure Co. Sure Co. Pure Co. Sure Co.

Net income from own operations P400,00 P200,000 P300,000 P150,000


Dividends paid 100,000 50,000 80,000 20,000

The intercompany gain is included in the net income of Sure Company. The equipment sold is expected to have a useful life of five
years from the date of sale.

The Non-controlling interests on December 31:

2011 2012 2011 2012

A. P226,400 P256,800 C. P226,000 P252,400

B. 226,400 253,200 D. 230,000 256,000

Question 19
During 2012, Pard Corp. sold goods to its 80% owned subsidiary, Seed Corp. At December 31, 2012, one-half (1/2) of these goods
were included in Seed's ending inventory. Reported 2012 selling expenses were P1,100,000 and P400,000 for Pard and Seed,
respectively. Pard's selling expenses included P50,000 in freight-out costs for goods sold to Seed. What amount of selling expenses
should be reported in Pard's consolidated income statement?

1,450,000
P1,475,000
P1,500,000
1,480,000

Question 20
Wright Corp. has several subsidiaries that are included in its consolidated financial statements. In its December 31, 2011, trial
balance, Wright had the following intercompany balances before eliminations:

Debit Credit

Current receivable due from Main Co. P32,000

Noncurrent receivable from Main 114,000

Cash advance to Corn Corp 6,000

Cash advance from King Co P15,000

Intercompany payable to King 101,000

In its December 31, 2011, consolidated balance sheet, what amount should Wright report as intercompany receivables?

0
146,000
P36,000
P152,000

Question 21
The Carly Company owns 75% of The Halley Company. The following figures are from their separate financial statements:

Carly: Trade receivables P1,040,000, including P30,000 due from Halley.

Halley: Trade receivables P215,000, including P40,000 due from Carly.

What figure should appear for trade receivables in Carly's consolidated statement of financial position?

P1,185,000
P1,255,000
P1,225,000
P1,215,000

Question 22
Clark Co. had the following transactions with affiliated parties during 2011.

Sales of P60,000 to Dean, Inc., with P20,000 gross profit. Dean had P15,000 of this inventory on hand at year end. Clark owns a
15% interest in Dean and does not exert significant influence.

Purchases of raw materials totaling P240,000 from Kent Corp., a wholly owned subsidiary. Kent's gross profit on the sale was
P48,000. Clark had P60,000 of this inventory remaining on December 31, 2011.

Before eliminating entries, Clark had consolidated current assets of P320,000. What amount should Clark report in its December 31,
2011, consolidated balance sheet for current assets?

P308,000
317,000
303,000
P320,000
Question 23
How should an entity account for the incomplete information in preparing the financial statements immediately after the acquisition?

Do not record the uncertain items until complete information is available.


Record the uncertain items at the carrying amount of the acquiree.
Record the uncertain items at a provisional amount measured at the date of acquisition.
Record a contra account to the investment account for the amounts involved.

Question 24
What is the proper treatment of measurement period adjustment?

Retroactively adjusted to retained earnings


Adjusted to other comprehensive income
Adjusted to profit or loss
Retroactively adjusted to goodwill or gain on bargain purchase

Question 25
A parent is exempted from preparing consolidated financial statements if all of the following conditions exist, except

The ultimate parent prepares consolidated financial statements that comply with IFRS.
The parent does not have any debt or equity instruments publicly traded.
The parent reports one class of share capital in the statement of financial position
The parent is wholly or partially owned and the owners do not object to the nonconsolidation.

Question 26
A parent entity controls 100% of an overseas subsidiary. Because of exchange controls, it is difficult to transfer funds out of the
country to the parent entity. How should the subsidiary be accounted for?

The subsidiary should be excluded from consolidation and accounted for as financial asset.
The subsidiary should be excluded from consolidation and the equity method should be used.
The subsidiary is not permitted to be excluded from consolidation because control is not lost.
The subsidiary should be excluded from consolidation and measured at cost.

Question 27
An entity acquired an investment in a subsidiary with the view to dispose of the investment within six months. The investment in the
subsidiary has been classified as held for sale. How should the investment in the subsidiary be treated in the financial statements?

Equity accounting should be used.


The subsidiary should be derecognized.
Acquisition accounting should be used.
The subsidiary should not be consolidated but IFRS 5 should be used.

Question 28
What is the initial measurement of the retained investment in subsidiary when control is lost?

Fair value at the beginning of the reporting period


Carrying amount at the date when control is lost
Fair value at the date when control is lost
Carrying amount at the beginning of the reporting period

Question 29
Penn Company, a manufacturing entity owns 75% of Sell Company. Sell Company, an investment entity, owns 60% of Vane
Company, an insurance entity. In Penn Company's consolidated financial statements, how should the investments in Sell Company
and Vane Company be accounted for?

Consolidated for Sell and equity method for Vane


Consolidated for both Sell and Vane
Equity method for both Sell and Vane
Equity method for Sell and consolidated for Vane

Question 30
Parent Company owns 80% of Subsidiary Company. During the current year, Parent sold goods with a 40% gross profit to
Subsidiary, Subsidiary sold all of these goods in the current year. For the consolidated financial statements, how should the
summation of income statement items be adjusted?

No adjustment is necessary.
Sales and cost of goods sold should be reduced by 80% of the intercompany sales.
Net income should be reduced by 80% of the gross profit of intercompany sales.
Sales and cost of goods sold should be reduced by the intercompany sales.

Question 31
Water Company owns 80% of the outstanding ordinary shares of Fire Company. At the end of the current year, Fire sold equipment
to Water Company at a price in excess of Fire's carrying amount, but less than the original cost. In the consolidated statement of
financial position at the current year-end, the carrying amount of the equipment should be reported at

Water's original cost


Water's original cost less 80% of Fire's recorded gain
Water's original cost less Fire's recorded gain
Fire's original cost

Question 32
Fortune Company owns 100% of Salem Company. At the beginning of current year, Fortune sold Salem delivery equipment at a
gain. Fortune had owned the equipment for two years and used a five-year straight line depreciation rate with no residual value.
Salem is using a three-year straight line depreciation rate with no residual value for the equipment. In the consolidated income
statement, Salem's recorded depreciation expense on the equipment for the current year should be decreased by

20% of the gain on sale


33 1/3% of the gain on sale
100% of the gain on sale
50% of the gain on sale

Intercompany Transactions Part 2 Quiz

Question 1

In 20x4, Parrot Company sold land to its subsidiary, Tree Corporation, for $12,000. It had a book value of $10,000. In the next year,
Tree sold the land for $18,000 to an unaffiliated firm.

Which of the following is correct?

A. No consolidation working paper entry was necessary in 20x4.

B. A consolidation working paper entry was required only if the subsidiary was less than 100% owned in 20x4.

C. A consolidation working paper entry is required each year until the land is sold outside the related parties.

D. A consolidated working paper entry was required only if the land was held for resale in 20x4.

The 20x4 unrealized gain

A. was deferred until 20x6.

B. was eliminated from consolidated net income by a working paper entry that credited land $2,000.

C. made consolidated net income $2,000 less than it would have been had the sale not occurred.

D. made consolidated net income $2,000 greater than it would have been had the sale not occurred.

Question 2
On January 1, 20x5, Eagle Corporation sold equipment with a book value of $40,000 and a 20-year remaining useful life to its
wholly-owned subsidiary, Rabbit Corporation, for $60,000. Both Eagle and Rabbit use the straight-line depreciation method,
assuming no salvage value. On December 31, 20x5, the separate company financial statements held the following balances
associated with the equipment:

Eagle Rabbit

Gain on sale of equipment $ 20,000

Depreciation expense $ 3,000

Equipment 60,000

Accumulated depreciation 3,000

A working paper entry to consolidate the financial statements of Eagle and Rabbit on December 31, 20x5 included a

credit to depreciation expense for $3,000.


debit to accumulated depreciation for $1,000.
debit to gain on sale of equipment for $19,000.
credit to gain on sale of equipment for $20,000.

Question 3
On January 2, 20x5 Kakapo Company sold a truck with book value of $45,000 to Flightless Corporation, its completely owned
subsidiary, for $60,000. The truck had a remaining useful life of three years with zero salvage value. Both firms use the straight-line
depreciation method, and assume no salvage value. If Kakapo failed to make year-end equity adjustments, Kakapo’s investment
in Flightless at December 31, 20x5 was

$10,000 too low.


$10,000 too high.
$15,000 too high.
$5,000 too high.

Question 4

On January 1, 20x3, Shrimp Corporation purchased a delivery truck with an expected useful life of five years. On January 1, 2005,
Shrimp sold the truck to Avocet Corporation and recorded the following journal entry:
Debit Credit

Cash 50,000

Accumulated depreciation 18,000

Truck 53,000
Gain on Sale of Truck 15,000

Avocet holds 60% of Shrimp. Shrimp reported net income of $55,000 in 20x5 and Avocet's separate net income (excludes interest
in Shrimp) for 20x5 was $98,000.

In preparing the consolidated financial statements for 20x5, the elimination entry for depreciation expense was a

A. debit for $5,000. C. debit for $15,000.

B. credit for $5,000. D. credit for $15,000.

In the consolidation working papers, the Truck account was

A. debited for $3,000. C. debited for $15,000.

B. credited for $3,000. D. credited for $15,000.

Consolidated net income for 20x5 was:

A. $121,000. B. $125,000. C. $131,000. D. $143,000.

The non-controlling interest income for 20x5 was

A. $18,000. B. $22,000. C. $23,000. D. $27,000.

Question 5
The Sox Co. owns 65% of the Gringo Co. On the last day of the accounting year, Gringo sold to Sox a non-current asset for
P200,000. The asset originally cost P500,000 and at the end of the reporting period its carrying amount in Gringo’s books was
P160,000. The group’s consolidated financial statement of financial position has been drafted without any adjustments in relation to
this noncurrent asset.

Under PAS 27, Consolidated and Separate Financial statements, what adjustments should be made to the consolidated statement
of financial position figures for non-current assets and retained earnings?

Increase by P300,000

Non-current Assets Retained Earnings

Increase by P300,000 Increase by P195,000

Reduce by P40,000 Reduce by P26,000

Reduce by P40,000 Reduce by P40,000

Increase by P300,000 Increase by P300,000

Question 6
Anton Co. owns a 100% interest in the ordinary shares of the Dial Co. On January 1, 20x3, Anton sold Dial an equipment that Dial
will use over a 5-year period. The asset sold at a P5,000 profit. In the consolidated statements, this profit will:
Be recognized when the asset is resold to outsider parties at the end of its period of use.

Be recognized over 5 years.


Be recognized in the year sale.
Not be recorded

Question 7
On January 1, 20x3, Palasyo Co. sold a machine for P900,000 to Single Corp, its wholly owned subsidiary. Palasyo paid
P1,100,000 for this machine, which had accumulated depreciation of P250,000. Palasyo estimated a P100,000 salvage value and
depreciated the machine on the straight line method over 20 years, a policy which Single continued. In Palasyo’s December 31,
20x3 consolidated statement of financial position, the machine should be included in cost and accumulated depreciation as:

Cost Accumulated depreciation Cost Accumulated depreciation


A. P1,100,000 P300,000 C. P900,000 P40,000
B. P1,100,000 P290,000 D. P850.000 P42,500

Question 8
Company Pilit owns 100% of the ordinary shares of Company Sigue-sigue. Company Pilit is constructing an asset for Company
Sigue-sigue that will be used in Company Sigue-sigue manufacturing operations over 5-year period. The asset was 50% complete
at the end of 20x6 and was completed on December 31, 20x7. Company Pilit is recording the construction under the percentage of
completion method. The asset was put to use by Company Sigue-sigue on January 1, 20x8. The profit on the asset was estimated
to be P50,000. Actual results complied to estimate. What amount of profit will appear on the consolidated financial statements?

20x6 20x7 20x8 20x9


A. 0 P 50,000 0 0
B. P25,000 25,000 0 0
C. 0 0 P 10,000 P10,000
D. 0 0 P50,000 0

Question 9
On January 1, 2014, Jana Company purchased 90% equity of Heller Company. On January 3, 2014, Heller sold equipment (with
original cost of P750,000 and carrying cost of P375,000 to Jana for P540,000. The equipment has a remaining life of three (3) years
and was depreciated using the straight line method by both companies. In Jana consolidated statement of financial position as of
December 31, 2014, the cost, accumulated depreciation and book value should be reported at:

Cost
Accumulated depreciation

Net Book value


A. P750,000 P500,000 P375,000
B. P375,000 P375,000 -0-
C. P750,000 P750,000 -0-
D. P750,000 P500,000 P250,000

Question 10
As of January 1, 2014, Posporo Company sold a warehouse with a book value of P80,000 and a 20 year remaining useful life to its
wholly owned subsidiary, Stan Corp for P120,000. Both Posporo and Stan use the straight-line depreciation method. On December
31, 2014, the separate company financial statements contained the following balances connected with the warehouse:

Posporo Stan
Gain on sale of warehouse P 40,000
Depreciation expense P 6,000
Warehouse 120,000
Accumulated depreciation 6,000

A working paper entry to consolidate the financial statements of Posporo and Stan on December 31, 2014 will include:

A debit to gain on sale of warehouse for P38,000.


A credit to depreciation expense for P6,000.
A debit to gain on sale of warehouse for P40,000.
A debit to accumulated depreciation for P2,000.

Question 11
On January 1, 2013, Puregold Company purchased 80 percent of the outstanding shares of Surebeat Company at a cost of
P1,000,000. On that date, Surebeat Company had P400,000 of Share Capital and P600,000 of retained earnings.

On July 1, 2013, Surebeat Company sold an equipment with a book value of P60,000 to Puregold for P80,000.

For 2013 and 2014, the results of their operations are:

2013 2014

Puregold Surebeat Puregold Surebeat

Net income from own operations P400,000 P200,000 P300,000 P150,000

Dividends paid 100,000 50,000 80,000 20,000

The intercompany gain is included in the net income of Surebeat Company. The equipment sold is expected to have a useful life of
five years from the date of sale.

The Non-controlling interests on December 31:

2013 2014

A. P226,400 P256,800

B. P226,400 P253,200

C. P226,000 P252,400

D. P230,000 P256,000
Question 12
Ground Parrot Company completely owns Heathlands Inc. On January 2, 20x5 Ground Parrot sold Heathlands machinery at its
book value of $30,000. Ground Parrot had the machinery two years before selling it and used a five-year straight-line depreciation
method, with zero salvage value. Heathlands will use a three-year straight-line method. In the 20x5 consolidated income statement,
the depreciation expense

increased by $4,000
required no adjustment.
decreased by $4,000.
increased by $30,000.

Question 13
In reference to the downstream or upstream sale of depreciable assets, which of the following statements is correct?

Gains and losses appear in the parent-company accounts in the year of sale and must be eliminated by the parent
company in determining its investment income under the equity method of accounting.
The initial effect of unrealized gains and losses from downstream sales of depreciable assets is different from the sale of
nondepreciable assets.
Upstream sales from the subsidiary to the parent company always result in unrealized gains or losses.
Gains, but not losses, appear in the parent-company accounts in the year of sale and must be eliminated by the parent company in
determining its investment income under the equity method of accounting.

Question 14
Falcon Corporation sold equipment to its 80%-owned subsidiary, Rodent Corp., on January 1, 20x5. Falcon sold the equipment for
$110,000 when its book value was $85,000 and it had a 5-year remaining useful life with no expected salvage value. Separate
balance sheets for Falcon and Rodent included the following equipment and accumulated depreciation amounts on December 31,
20x5:

Falcon Rodent

Equipment $ 750,000 $300,000

Less: Accumulated depreciation ( 200,000) ( 50,000)

Equipment-net $ 550,000 $250,000

Consolidated amounts for equipment and accumulated depreciation at December 31, 20x5 were respectively

$1,025,000 and $245,000.


$1,025,000 and $245,000.
$1,050,000 and $250,000.
$1,025,000 and $250,000.

Question 15

Peregrine Corporation acquired a 90% interest in Cliff Corporation in 20x4 at a time when Cliff’s book values and fair values were
equal to one another. On January 1, 20x5, Cliff sold a truck with a $45,000 book value to Peregrine for $90,000. Peregrine is
depreciating the truck over 10 years using the straight-line method. Separate incomes for Peregrine and Cliff for 20x5 were as
follows:

Peregrine Cliff

Sales $1,800,000 $1,050,000

Gain on sale of truck 45,000

Cost of Goods Sold ( 750,000) ( 285,000)

Depreciation expense ( 450,000) ( 135,000)

Other expenses ( 180,000) ( 450,000)

Separate incomes $420,000 $225,000

Peregrine’s investment income from Cliff for 20x5 was

$162,000.
$166,050.
$202,500
$161,550.

Question 16
Kestrel Company acquired an 80% interest in Reptile Corporation on January 1, 20x4. On January 1, 20x5, Reptile sold a building
with a book value of $50,000 to Kestrel for $80,000. The building had a remaining useful life of ten years and no salvage value. The
separate statements of financial position of Kestrel and Reptile on December 31, 20x5 included the following balances:

Kestrel Reptile
Buildings $400,000 $250,000

Accumulated Depreciation - Buildings 120,000 75,000

The consolidated amounts for Buildings and Accumulated Depreciation - Buildings that appeared, respectively, on the statement of
financial position at December 31, 20x5, were

$620,000 and $192,000.


$620,000 and $195,000.
$650,000 and $192,000.
$650,000 and $195,000.

Question 17
Pigeon Corporation purchased land from its 60%-owned subsidiary, Seed Inc., in 20x3 at a cost $30,000 greater than Seed’s book
value. In 20x5, Pigeon sold the land to an outside entity for $40,000 more than Pigeon’s book value. The 20x5 consolidated income
statement reported a gain on the sale of land of

$70,000.
$58,000.
$42,000.
$40,000.

Question 18
Pied Imperial-Pigeon Corporation acquired a 90% interest in Offshore Corporation in 20x3 when Offshore’ book values were
equivalent to fair values. Offshore sold equipment with a book value of $80,000 to Pied Imperial-Pigeon for $130,000 on January 1,
20x5. Pied Imperial-Pigeon is fully depreciating the equipment over a 4-year period by using the straight-line method. Offshore’
reported net income for 20x5 was $320,000. Pied Imperial-Pigeon’s 20x5 net income from Offshore was

$288,000.
$254,250.
$250,500.
$249,250.

Question 19
Lorikeet Corporation acquired a 80% interest in Nectar Corporation on January 1, 20x1 at a cost equal to book value and fair value.
In the same year Nectar sold land costing $30,000 to Lorikeet for $50,000 On July 1, 20x6, Lorikeet sold the land to an unrelated
party for $110,000. What was the gain on the consolidated income statement?

$64,000.
$60,000.
$48,000.
$80,000.

Question 20
On January 1, 20x5 Rainforest Co. recorded a $30,000 profit on the upstream sale of some equipment that had a remaining four-
year life under the straight-line depreciation method. The effect of this transaction on the amount recorded in 20x5 by the parent
company Wompoo as its investment income in the Rainforest was

an increase of $22,500 if the Rainforest was wholly owned.


an increase of $30,000 if the Rainforest was wholly owned.
a decrease of $27,000 if the Rainforest was 90% owned.
a decrease of $18,000 if the Rainforest was 80% owned.

Question 21
Swift Parrot Corporation acquired a 60% interest in Berries Corp. on January 1, 20x5, when Berries’s book values and fair values
were equivalent. On January 1, 2005, Berries sold a building with a book value of $600,000 to Swift Parrot for $700,000. The
building had a remaining life of 10 years, no salvage value, and was depreciated by the straight-line method. Berries reported net
income of $2,000,000 for 20x5. What was the noncontrolling interest for 20x5?

$710,000.
$900,000.
$764,000.
$800,000.

Question 22
ACE Company acquired the net assets of CRC Corporation on January 1, 2017. Since the parties cannot agree on the definite
value of the company in terms of potential future earnings, they agreed to include in the purchase agreement a provision for
contingent consideration. whereby the acquirer will pay an additional cash payment on January 1, 2019 equal to twice the amount
by which average earnings of CRC exceed P250,000 per year, prior to January 1, 2019. Net income was P500,000 in 2017 and
P600,000 in 2018. Assume that the liabilities recorded on January 1, 2017, include an estimated contingent liability amounting to
P400,000.

What was the entry made by ACE in January 1, 2019?

A. Liability from contingent consideration400,000

Cash 400,000

B. Goodwill 200,000

Liability from contingent consideration 400,000


Cash 600,000
C. Liability from contingent consideration 400,000

Loss on contingent consideration 200,000

Cash 600,000

D. Goodwill 600,000
Cash 600,000

Question 23

Income statement information for the year 2012 for Perfect Corporation and its 60% owned subsidiary, Seven Corporation, is as
follows:

Perfect Seven

Sales P900,000 P350,000

Cost of Sales 400,000 250,000

Gross profit P500,000 P 100,000

Operating expenses 250,000 50,000

Seven’s net income P 50,000

Perfect’s Separate incomes P 250,000

Intercompany sales for 2012 are upstream (from Seven to Perfect) and total P100,000. Perfect's December 31, 2011 and
December 31, 2012 inventories contain unrealized profits of P5,000 and P10,000, respectively.

The Consolidated sales for 2012:

A. P900,000 C. P1,190,000

B. 1,150,000 D. 1,250,000

The Consolidated cost of sales for 2012:

A. P545,000 C. P555,000

B. 550,000 D. 560,000

The Profit attributable to Equity Holders of Parent or CNI Contributable to Controlling Interests for 2012:

A. P277,000 C. P282,000

B. 280,000 D. 305,000

Question 24

Parry Corporation owns an 80% interest in Starry Corporation acquired several years ago. Starry regularly sells merchandise to its
parent at 125% of Starry's cost. Gross profit data of Parry and Starry for the year 2012 are as follows:

Parry Starry

Sales P1,000,000 P800,000

Cost of Sales 800,000 640,000

Gross profit P 200,000 P 160,000


During 2012, Parry purchased inventory items from Starry at a transfer price of P400.000. Parry's December 31, 2011 and 2012
inventories included goods acquired from Starry of P100,000 and P125,000, respectively.

The Consolidated sales of Parry Corporation and subsidiary for 2012 were:
A. P1,800,000 C. P1,400,000

B. 1,425,000 D. 1,240,000
Using the same information in No. 137, the Unrealized profits in the year-end 2011 and 2012 inventories were:

A. P100,000 and P125,000 respectively.

B. P800,000 and P100,000 respectively.

C. P20,000 and P25,000 respectively.

D. P16,000 and P20,000 respectively.

Using the same information in No. 137, the Consolidated cost of goods sold at Parry and subsidiary for 2012 was:

A. P1,024,000 C. P1,052,800

B. 1,045,000 D. 1,056,000

Question 25
Power Co. is a manufacturer and Slack Co., its 100%-owned subsidiary, is a retailer. The companies are vertically integrated. Thus,
Slack purchases all of its inventory from Power. On January 1, 2012. Slack’s inventory was P30,000. For the year ended December
31, 2012, its purchases were P150,000, and its cost of sales was P166,500. Power's sales to Slack reflect a 50% markup on cost.
Slack then resells the goods to outside entitles at a 100% markup on cost. At what amount should the intercompany inventory
purchased from Power be reported in the consolidated balance sheet at December 31, 2012?

46,000
9,000
P13,500
P3,000

Question 26
The Lakers Company owns 75% of The Viking Company. On December 31, 2012, the last day of the accounting period, Vikings
sold to Lakers a noncurrent asset for P200,000. The asset's original cost was P500,000 and on December 31, 2012 its carrying
amount in Viking's books was P160,000. The group's consolidated statement of financial position has been drafted without any
adjustments in relation to this non-current asset.

What adjustments should be made to the consolidated statement of financial position figures for retained earnings and non-
controlling interest?

Retained earnings Non-controlling interest

A. Increase by P225,000 Increase by P75,000

B. Increase by P300,000 No change

C. Reduce by P30,000 Reduce by P10,000

D. Reduce by P40,000 No change

Question 27
On January 1, 2012, Jan Company purchased 90% equity of Jo Company. On January 3, 2012, Jo sold equipment (with original
cost of P750,000 and carrying cost of P375,000) to Jan for P540,000. The equipment have a remaining life of three (3) years and
was depreciated using the straight-line method by both companies. In Jan consolidated balance sheet as of December 31, 2012,
the cost, accumulated depreciation and book value should be reported at:

Cost Accumulated depreciation Net book Value

A. P750,000 P500,000 P375,000

B. 375,000 375,000 -0-

C. 750,000 750,000 -0-

D. 750,000 500,000 250,000

Question 28
As January 1, 2012, Par Corp. sold a warehouse with a book value of P80,000 and a 20-year remaining useful life to its wholly-
owned subsidiary, Strata Corporation, for P120,000. Both Par and Strata use the straight-line depreciation method. On December
31, 2012, the separate company financial statements contained the following balances connected with the warehouse:

Par Strata

Gain on sale of warehouse P40,000

Depreciation expense P 6,000

Warehouse 120,000
Accumulated depreciation 6,000

A working paper entry to consolidate the financial statements of Par and Strata on December 31, 2012 will include:

A debit to gain on sale of warehouse for P40,000.


A debit to accumulated depreciation for P2,000.
A credit to depreciation expense for P6,000.
A debit to gain on sale of warehouse for P38,000.

Question 29
Pero Corporation owns a 70% interest in Sweet Corporation, acquired several years ago at book value. On December 31, 2011,
Sweet mailed a check for P10,000 to Pero in part payment of a P20,000 account with Pero. Pero had not received the check when
its books were closed on December 31. Pero Corporation had accounts receivable of P150,000 (including the P20,000 from Sweet)
and Sweet had accounts receivable at P220,000 at year-end. In the consolidated balance sheet of Pero Corporation and Subsidiary
at December 31, 2011 accounts receivable will be shown in the amount of:

304,000
360,000
P370,000
P350,000

Question 30
Cobb Company's current receivables from affiliated companies at December 31, 2011 are (1) a P75,000 cash advance to Hill
Corporation (Cobb owns 30% of the voting stock of Hill and accounts for the investment by the equity method), (2) a receivable of
P260,000 from Vick Corporation for administrative and selling services (Vick is 100%-owned by Cobb and is included in Cobb's
consolidated financial statements), and (3) a receivable of P200,000 from Ward Corporation for merchandise sales on credit (Ward
is a 90%-owned, unconsolidated subsidiary of Cobb accounted for by the equity method). In the current assets section of its
December 31, 2011 consolidated balance sheet, Cob should report accounts receivable from investees in the amount of:

535,000
255,000
P180,000
P275,000

Question 31

On January 1, 2013, P Company purchased 80 percent of the outstanding shares of S Company by paying P700,000. On that date,
S Company had P300,000 capital stock and P500,000 of retained earnings. An undervalued asset attributable to building
amounting to P75,000 with a remaining life of 25 years. All other assets and liabilities of S Company had book value approximated
their fair market value.

On January 1, 2014, P's common stock and retained earnings amounted to P1,000,000 and P800,000, respectively, white S
Company's retained earnings is P600,000.

The 2014 net income and dividends using cost (or initial value) model was as follows:

Net income Dividends

P Company P340,000 P100,000

S Company 150,000 P50,000

On April 1, 2014, S Company sold equipment with a book value of P30,000 to P Company for P60,000. The gain on the sale is
included in the net income of S Company indicated above. The equipment is expected to have a remaining useful life of five years
from the date of the sale.

On September 30, 2014. P Company sold machinery with a book value of P40,000 to S Company for P75,000. The gain on the sale
is also included in the net income of P Company indicated above. The machinery is expected to last for ten (10) years from the date
of sale.

the Non-controlling interest in Net Income for 2014:

A. P30,000 C. P24,900

B. P25,500 D. P24,300

the Profit attributable to equity holders of parent (Parent's Interest/Controlling Interest in Profit) for 2014:

A. P356,500 C. P363,075

B. P362,200 D. P386,500

the Consolidated/Group Net Income for 2014:

A. P356,500 C. P363,075
B. P362,200 D. P387,375

the consolidated stockholders' equity on December 31, 2014:


B

A. P2,040,000 C. P2,358,375

B. P2,349,375 D. P2.375,975

Quiz in Intercompany Transactions Part 3

Question 1

The separate incomes (which do not include investment income) of Pell Corporation and Sell Corporation, its 80% owned subsidiary,
for 2012 were determined as follows:

Pell Shell

Sales P400,000 P100,000

Less Cost of Sales 200,000 60,000

Gross profit P200,000 P 40,000

Other expenses 100,000 30,000

Separate incomes P 100,000 P 10,000

During 2012 Pell sold merchandise that cost P20,000 to Sell for P40,000, and at December 31, 2012 half of these inventory items
remained unsold by Sell.

The Non-controlling interest in net income for 2012:

A. P 0 C. P 8,000

B. 2,000 D. 10,000

The Consolidated sales for 2012:

A. P500,000 C. P460,00

B. 480,000 D. 400,000

The Consolidated cost of sales for 2012:

A. P230,000 C. P270,000

B. 248,000 D. 300,000

The Profit attributable to Equity Holders of Parent or CNI Contributable to controlling Interests for 2012:

A. P108,000 C. P98,000

B. 100,000 D. 80,000

Question 2

On January 1, 2013, Par Company purchased 80% of the outstanding shares of Sub Company by paying P340,000, the Sub
Company's common stock and retained earnings on this date amounted to P150,000 and P230,000 respectively. Also on this date,
an equipment is undervalued by P20,000 with a remaining life of 10 years.

On January 1, 2015, Sub Company had P150,000 of capital stock and P300,000 or retained earnings. Also on the same date, Par
Company had P1,000,000 of capital stock and P700,000 of retained earnings.
During the year, Par Company sold merchandise to Sub for P60,000 and in turn, purchased P40,000 from Sub Company. Inter-
company sales of merchandise were made at the following gross profit rates:

Sales made by parent 25% based on cost

Sales made by subsidiary 20% based on sales

On December 31, 2015, 30% of all inter-company sales remain in the ending inventory of the purchasing affiliate.

The beginning inventory of Par Company includes P2,500 worth of merchandise acquired from Sub Company on which Sub
Company reported a profit of P1,000. While, the beginning inventory of Sub also includes P3,000 of merchandise acquired from Par
Company at 35% mark-up.
Using cost method the following selected results of operations for 2015 were as follows:

Par Company Sub Company

Dividends paid P 60,000 P10,000

Net income from own operations P100,000 P30,000

Add: Dividend income 8,000

Net income P108,000 P30,000

the Non-controlling Interest in Net Income for 2015, should be:

A. P6,280 C. P5,720

B. P6,120 D. P5,320

the consolidated stockholders' equity on December 31, 2015 using proportionate basis (or partial goodwill approach):

A. P1,911,000 C. P1,905,920

B. P1,906,000 D. P1,740,000

the consolidated stockholders' equity on December 31, 2015 using full fair value basis (or full-goodwill approach) should
be:

A. P1,911,000 C. P1,905,920

B. P1,906,000 D. P1,740,000

Question 3

Silver Corporation is a 90% owned subsidiary of Proto Corporation acquired several years ago at book value equal to fair value. For
the years 2011 and 2012, Proto and Silver report the following:

2011 2012

Proto's separate income P300,000 P400,000

Silver's net income 80,000 60,000

The only intercompany transaction between Proto and Silver during 2011 and 2012 was the January 1, 2011 sale of land. The land
had a book value of P20,000 and was sold intercompany for P30,000, its appraised value at the time of sale.

If the land was sold by Proto to Silver (downstream sales) and that Silver still owns the land at December 31, 2012, compute the
Profit Attributable to Equity Holders of Parent for 2011 and 2012:

2011 2012 2011 2012

A. P363,000 P454,000 C. P372,000 P460,000

B. 362,000 454,000 D. P362,000 460,000

the Consolidated/group net income for 2011 and 2012:

2011 2012 2011 2012

A. P362,000 P454,000 C. P370,000 P460,000

B. 380,000 460,000 D. P372,000 460,000


Using the same information in No. 8, except that the land was sold by Silver to Proto (upstream sales) and Proto still owns the land
at December 31, 2008, compute the Profit Attributable to Equity Holders of Parent or CNI Attributable to Controlling Interests for
2011 and 2012:

2011 2012 2011 2012

A. P363,000 P454,000 C.P370,000 P460,000

B. 362,000 454,000 D.P363,000 460,000


Using the same information in No. 10, the Consolidated/group net income for 2011 and 2012:

2011 2012 2011 2012

A. P362,000 P454,000 C. P370,000 P460,000

B. 380,000 460,000 D.P372,000 460,000

Question 4
Scroll, Inc. a wholly owned subsidiary of Pirn, Inc. began operations on January 1, 2012. The following information is from the
condensed 2012 income statements of Pirn and Scroll:

Pirn Scroll

Sales to Scroll P100,000 P -

Sales to others 400,000 300,000

P500,000 P300,000

Cost of goods sold:

Acquired from Pirn - 80,000

Acquired from others 350,000 190,000

Gross profit P150,000 P30,000

Depreciation 40,000 10,000

Other expenses 60,000 15,000

Income from operations P50,000 P 5,000

Gain on sales of equipment to

Scroll 12,000 -

Income before taxes P 62,000 P 5,000

Additional Information:

Sales by Pirn to Scroll are made on the same terms as those made to third parties.
Equipment purchased by Scroll from Pirn for P36,000 on January 1, 2012 is depreciated using the straight-line method over four-
years.

For purposes of consolidation on December 31, 2012, what amount of intercompany profit that should be eliminated from Scroll's
inventory in the consolidated financial statements?

A. P6,000 C. P20,000

B. P10,000 D. P30,000

the amount of depreciation expense in the consolidated F/S?

A. P44,000 C. P50,000

B. 47,000 D. P53,000

Question 5
Justings Co. owned 80% of Evana Corp. During 2011, Justings sold to Evana land with a book value of P48,000. The selling price
was P70,000. In its accounting records, Justings should:

Not recognize a gain on the sale of the land since it was made to a related party
Recognize a gain of P22,000
Defer recognition of the gain until Evana sells the land to a third party
Recognize a gain of P17,600

Question 6
Par Corp. owns 60% of Sub Corp.'s outstanding capital stock. On May I, 2011, Par advanced Sub P70,000 in cash, which was still
outstanding at December 31, 2012. What portion of this advance should be eliminated in the preparation of the December 31, 2012
consolidated balance sheet?

42,000
0
P28,000
P70,000

Question 7
At December 31,2012, Grey, Inc. owned 90% of Winn Corp., a consolidated subsidiary, and 20% of Carr Corp., an investee over
which Grey cannot exercise significant influence. On the same date, Grey had receivables of P300,000 from Winn and P200,000
from Carr. In its December 31, 2012 consolidated balance sheet, Grey should report accounts receivable from affiliates of:

200,000
P230,000
340,000
P500,000

Question 8
Dean, Inc. owns 100% of Roy Corporation, a consolidated subsidiary, and 80% of Wall, Inc., an unconsolidated subsidiary at 12/31.
On the same date, Dean has receivables of P200,000 from Roy and P175,000 from Wall. In its 12/31 consolidated balance sheet,
Dean should report accounts receivable from investees at

P0
P175,000
235,000
35,000

Question 9
Denim Co. merged into Kraft Corp. on July 1, 2013. In exchange for the net assets at fair market value of Denim Co. amounting to
P696,450, Kraft issued 68,000 common shares at P9 par value with a market price of P12 per share

Out of pocket costs of the combination were as follows:

Legal fees for the contract of business combination P35,600

Audit fee for SEC registration of stock issue 90,000

Printing costs of stock certificates 14,500

Broker's fee 23,600

Accountant's fee for pre-acquisition audit 80,000

Other direct cost of acquisition 75,000

General and allocated expenses 43,000

Listing fees in issuing new shares 36,000

Denim will pay an additional cash consideration of P455,000 in the event that Kraft's net income will be equal or greater than
P950,000 for the period ended December 31, 2013. At acquisition date, there is a high probability of reaching the target net income
and the fair value of the additional consideration was determined to be P195,000.

Actual net income for the period ended December 31, 2013 amounted to P1,250,000. The additional cash consideration was paid.

What is the amount of goodwill to be recognized in the statement of financial position as of December 31, 2013?

A. P295,450

B. P308,500

C. P314,550

D. P326,550

What is the amount of expense to be recognized in the statement of comprehensive income for the year ended December
31, 2013?

A. P257,200

B. P517,200

C. P307,400

D. P412,500

Question 10
On October 1, 2013, Winner Company acquired all the assets and assumed all the liabilities of Getter Company by issuing 20,000
shares with a fair value of P67.5 per share and an obligation to pay a contingent consideration with a fair value of P750,000

In addition, Winner paid the following acquisition related costs:

Legal fees P105,600

Audit fee for SEC registration of stock issue 320,400

Costs of stock certificates 35,000

Broker's fee 49,000

Other direct cost of acquisition 50,000

General and allocated expenses 14,000

The Statement of Financial Position as of September 30, 2013 of Winner and Getter, together with the fair market value of the
assets and liabilities are presented below:

Winner Getter

Book Value Fair Value Book Value Fair Value

Cash P640,000 P640,000 P45,000 P45,000

Account receivable 360,000 335,000 70,000 54,000

Inventories 475,000 390,000 87,000 78,000

Prepaid expenses 25,000 13,500 5,000

Land 2,000,000 2,900,000 900,000 1,550,000

Building 800,000 900,000 723,000 768,000

Equipment 700,000 585,000 361,500 360,000

Goodwill 300,000

Total assets P5,000,000 P5,750,000 P2,500,000 P2,860,000

Accounts payable 312,500 312,500 200,000 200,000

Notes payable 937,500 980,000 700,000 765,000

Capital stock, 50 par 2,000,000 850,000

Additional paid in capital1,000,000 400,000

Retained earnings 750,000 350,000

Total equities P5,000,000 P2,500,000

Compute for the balances that will be shown on the October 1, 2013 statement of financial position of the surviving company:

Retained earnings

A. P480,000

B. P540,000

C. P526,000

D. P475,000

Total assets

A. P7,015,000

B. P6,980,000

C. P7,118,000

D. P7,491,000

Question 11
The Statement of Financial Position of Luster Corporation on June 30, 2013 is presented below:

Currents assets P32,500


Land 220,000

Building 110,000

Equipment 87,500

Total assets P450,000

Liabilities 87,500

Capital stock, 5 par 150,000

Additional paid in capital137,500

Retained earnings 75,000

Total equities P450,000

All the assets and liabilities of Luster assumed to approximate their fair values except for land and building. It is estimated that the
land has a fair value of P350,000 and the fair value of the building increased by P80,000.

Kernel Corporation acquired 80% of Luster's capital stock for P500,000.

Assuming the consideration paid includes control premium of P142,000, how much is the goodwill/ (gain on acquisition)
on the consolidated financial statement?

A. P60,000

B. P48,000

C. P42,000

D. P50,000

Assuming the consideration paid excludes control premium of P23,000 and the fair value of the non-controlling interest is
P122,750, how much

A. P78,250

B. P73,250

C. P69,500

D. P74,750

Assuming the consideration paid includes control premium of P37,000, how much is the goodwill/ (gain on acquisition) on
the consolidated financial statement? (Full)

A. P43,250

B. P73,250

C. P56,750

D. P68,350

Question 12
On January 2, 2012, D Corporation purchased 80% of the outstanding shares of C Company for P4,750,000. At that date, C had
P4,000,000 of ordinary shares outstanding and retained earnings of P1,600,000.

C's equipment with a remaining life of 5 years had a book value of P2,250,000 and a fair value of P2,630,000 C's remaining assets
had book values equal to their fair values.
All intangibles except goodwill are expected to have remaining lives of 8 years.
The income and dividend figures for both D and C are as follows: Net income of D in 2012 is P900,000; 2013 is P1,100,000. Net
income of C in 2012 is P340,000 ; 2013 is P510,000
Dividends of D in 2012 is P220,000; 2013 is P390,000. Dividends of C in 2012 is P70,000 ; 2013 is P130,000.
D's retained earnings balance at the date of acquisition was P3,450,000.

How much is the consolidated retained earnings attributable to controlling interest in 2013?
A. P5,272,400

B. P5,333,200

C. P5,238,400

D. P5,232,400

Share of D Corporation in the adjusted and undistributed earnings of C Company in 2012


A. P211,200

B. P155,200

C. P216,000

D. P182,400

How much is the consolidated profit in 2013?

A. P1,343,200

B. P1,438,000

C. P1,430,000

D. P1,464,000

How much is the non-controlling interest in net assets in 2013?

A. P1,295,600

B. P1,250,000

C. P1,302,400

D. P1,289,500

Question 13
Pure Corporation acquired an 80% interest in Sincere Company on January 2, 2012 for P2,520,000. On this date, the share capital
and retained earnings of the two companies follow:

Pure Corp. Sincere Co.

Share Capital P6,000,000 P2,250,000

Retained Earnings 3,000,000 450,000

On January 2, 2012, the assets and liabilities of Sincere Co. were stated at their fair values except for machinery which is
undervalued by P225,000 (remaining life is 3 years). On September 30, 2012, Sincere sold merchandise to Pure at an inter-
company profit of P150,000, 25% was still unsold at year-end. Likewise, on October 1, 2013, Sincere purchased merchandise from
Pure for P3,600,000. The selling affiliate included a 20% mark-up on cost on this sale. Only 75% of these purchases had been sold
to unrelated parties as of December 31, 2013. As of December 31, 2013, goodwill was determined to be impaired by P60,000.

The following is the summary of the 2013 transactions of the affiliated companies:

Pure Corp. Sincere Co.

Net Income P1,500,000 P600,000

Dividends declared and paid 600,000 180,000

On the 2013 consolidated financial statements, how much would be the:

Net income attributable to Parent

A. P1,638,000

B. P1,708,500

C. P1,608,000

D. P1,686,000

Non-controlling interest in net income

A. P70,500

B. P100,500

C. P82,500

D. P85,500

Question 14
On January 2, 2012, Power Company acquired 90% of the outstanding shares of Solar Inc. at book value. During 2012 and 2013,
intercompany sales amounted to P2,000,000 and P4,000,000, respectively. Power Company consistently recognized a 25% mark-
up based on cost while Solar Inc. had a 25% gross profit on sales. The inventories of the buying affiliate, which all came from inter-
company transactions show:

December 31, 2012 December 31, 2013

Power P240,000 P160,000

Solar 100,000 40,000

On October 1, 2012, Solar Inc., purchased a piece of land costing P1,000,000 from Power Company for P1,500,000. On December
1, 2013, Solar Inc., sold this land to unrelated party for P1,500,000. On the other hand, on July 1, 2013, Solar Inc., sold a used
photo-copier with a carrying value of P60,000 and remaining life of 3 years to Power Company for P42,000.

Statement of Comprehensive Income for the two companies for the year 2013 follow:

Power Company Solar Inc.

Sales P25,000,000 P14,000,000

Cost of Sales (15,000,000) ( 8,400,000)

Gross Profit P10,000,000 P 5,600,000

Operating Expenses (6,000,000) ( 3,800,000)

Operating Profit P 4,000,000 P 1,800,000

Loss on Sale of Office Equipment ( 18,000)

Dividend Revenue 40,000

Net income P 4,000,000 P 1,822,000

Compute the following amounts for/as of December 31, 2013

Consolidated Gross Profit

A. P19,632,000

B. P15,712,000

C. P15,632,000

D. P15,584,000

Consolidated Net Income attributable to Parent

A. P6,183,300

B. P6,369,000

C. P6,169,800

D. P6,191,300

Non-controlling interest in Net Income

A. P189,700

B. P185,700

C. P188,200

D. P184,200

Consolidated Operating Expense

A. P9,800,000

B. P9,788,000

C. P9,803,000

D. P9,789,500

You might also like